*NURSING > TEST BANKS > PAeasy GI Questions and answers, 100% rated A+.Test Bank Questions (All)
PAeasy GI Questions and answers, 100% rated A+.Test Bank Questions Ulcerative Colitis appearance on colonoscopy - ✔✔Crypt abscesses and if toxic megacolon present loss of haustra and blood ... vessels A 63 year-old woman complains of fatigue, loss of appetite, a sore-red tongue, paresthesias of her feet and hands, and unsteadiness of her gait. Which of the following tests is be used to confirm the patient's suspected diagnosis? A Anti-intrinsic factor antibodies B Antiparietal cell antibodies C Coomb's test D Schilling Test E Serum folate levels - ✔✔Anti-intrinsic factor antibodies A diagnosis of pernicious anemia (PA) can be supported through the presence of anti-intrinsic factor antibodies. Antiparietal (B) cell antibodies aren't associated with the development of PA. The Coomb's test (C) is used in the evaluation of hemolytic anemias. Serum folate (D) levels are beneficial in evaluating macrocytic anemias, but will not establish a diagnosis of PA. The Schilling test (D) was once commonly used to diagnosis PA, but is no longer available due to lack of available radiolabeled human intrinsic factor. A 34-year-old male presents to the primary care office with a complaint of heartburn that has been present for three months. He has symptoms two to three times a week, which occurs about 30 minutes after eating. He has tried over-the-counter antacids and they were helping to relieve his symptoms for a few months, but they are not working well now. He denies dysphagia, odynophagia, or weight loss. You decide to treat him with a proton pump inhibitor at this visit, and he achieves good symptomatic relief with this therapy. What length of therapy is appropriate in this patient? A Two to four weeks B Eight to twelve weeks C Four to six months D One year E Continue indefinitely - ✔✔If a patient achieves good symptomatic relief with a course of an empiric, once-daily proton pump inhibitor, therapy may be discontinued after eight to twelve weeks. A 34-year-old male presents to the primary care office with a complaint of heartburn that has been present for three months. He has symptoms two to three times a week, which occurs about 30 minutes after eating. He has tried over-the-counter antacids and they were helping to relieve his symptoms for a few months, but they are not working well now. He denies dysphagia, odynophagia, or weight loss. You decide to treat him with a proton pump inhibitor at this visit, and he achieves good symptomatic relief with this therapy. What length of therapy is appropriate in this patient? A Two to four weeks B Eight to twelve weeks C Four to six months D One year E Continue indefinitely - ✔✔If a patient achieves good symptomatic relief with a course of an empiric, once-daily proton pump inhibitor, therapy may be discontinued after eight to twelve weeks. A 34-year-old male presents to the primary care office with a complaint of heartburn that has been present for three months. He has symptoms two to three times a week, which occurs about 30 minutes after eating. He has tried over-the-counter antacids and they were helping to relieve his symptoms for a few months, but they are not working well now. He denies dysphagia, odynophagia, or weight loss. You decide to treat him with a proton pump inhibitor at this visit, and he achieves good symptomatic relief with this therapy. What length of therapy is appropriate in this patient? A Two to four weeks B Eight to twelve weeks C Four to six months D One year E Continue indefinitely - ✔✔If a patient achieves good symptomatic relief with a course of an empiric, once-daily proton pump inhibitor, therapy may be discontinued after eight to twelve weeks. A 34-year-old male presents to the primary care office with a complaint of heartburn that has been present for three months. He has symptoms two to three times a week, which occurs about 30 minutes after eating. He has tried over-the-counter antacids and they were helping to relieve his symptoms for a few months, but they are not working well now. He denies dysphagia, odynophagia, or weight loss. You decide to treat him with a proton pump inhibitor at this visit, and he achieves good symptomatic relief with this therapy. What length of therapy is appropriate in this patient? A Two to four weeks B Eight to twelve weeks C Four to six months D One year E Continue indefinitely - ✔✔If a patient achieves good symptomatic relief with a course of an empiric, once-daily proton pump inhibitor, therapy may be discontinued after eight to twelve weeks. A 34-year-old male presents to the primary care office with a complaint of heartburn that has been present for three months. He has symptoms two to three times a week, which occurs about 30 minutes after eating. He has tried over-the-counter antacids and they were helping to relieve his symptoms for a few months, but they are not working well now. He denies dysphagia, odynophagia, or weight loss. You decide to treat him with a proton pump inhibitor at this visit, and he achieves good symptomatic relief with this therapy. What length of therapy is appropriate in this patient? A Two to four weeks B Eight to twelve weeks C Four to six months D One year E Continue indefinitely - ✔✔If a patient achieves good symptomatic relief with a course of an empiric, once-daily proton pump inhibitor, therapy may be discontinued after eight to twelve weeks. A 34-year-old male presents to the primary care office with a complaint of heartburn that has been present for three months. He has symptoms two to three times a week, which occurs about 30 minutes after eating. He has tried over-the-counter antacids and they were helping to relieve his symptoms for a few months, but they are not working well now. He denies dysphagia, odynophagia, or weight loss. You decide to treat him with a proton pump inhibitor at this visit, and he achieves good symptomatic relief with this therapy. What length of therapy is appropriate in this patient? A Two to four weeks B Eight to twelve weeks C Four to six months D One year E Continue indefinitely - ✔✔If a patient achieves good symptomatic relief with a course of an empiric, once-daily proton pump inhibitor, therapy may be discontinued after eight to twelve weeks. A 34-year-old male presents to the primary care office with a complaint of heartburn that has been present for three months. He has symptoms two to three times a week, which occurs about 30 minutes after eating. He has tried over-the-counter antacids and they were helping to relieve his symptoms for a few months, but they are not working well now. He denies dysphagia, odynophagia, or weight loss. You decide to treat him with a proton pump inhibitor at this visit, and he achieves good symptomatic relief with this therapy. What length of therapy is appropriate in this patient? A Two to four weeks B Eight to twelve weeks C Four to six months D One year E Continue indefinitely - ✔✔If a patient achieves good symptomatic relief with a course of an empiric, once-daily proton pump inhibitor, therapy may be discontinued after eight to twelve weeks. A 34-year-old male presents to the primary care office with a complaint of heartburn that has been present for three months. He has symptoms two to three times a week, which occurs about 30 minutes after eating. He has tried over-the-counter antacids and they were helping to relieve his symptoms for a few months, but they are not working well now. He denies dysphagia, odynophagia, or weight loss. You decide to treat him with a proton pump inhibitor at this visit, and he achieves good symptomatic relief with this therapy. What length of therapy is appropriate in this patient? A Two to four weeks B Eight to twelve weeks C Four to six months D One year E Continue indefinitely - ✔✔If a patient achieves good symptomatic relief with a course of an empiric, once-daily proton pump inhibitor, therapy may be discontinued after eight to twelve weeks. A 34-year-old male presents to the primary care office with a complaint of heartburn that has been present for three months. He has symptoms two to three times a week, which occurs about 30 minutes after eating. He has tried over-the-counter antacids and they were helping to relieve his symptoms for a few months, but they are not working well now. He denies dysphagia, odynophagia, or weight loss. You decide to treat him with a proton pump inhibitor at this visit, and he achieves good symptomatic relief with this therapy. What length of therapy is appropriate in this patient? A Two to four weeks B Eight to twelve weeks C Four to six months D One year E Continue indefinitely - ✔✔If a patient achieves good symptomatic relief with a course of an empiric, once-daily proton pump inhibitor, therapy may be discontinued after eight to twelve weeks. A 34-year-old male presents to the primary care office with a complaint of heartburn that has been present for three months. He has symptoms two to three times a week, which occurs about 30 minutes after eating. He has tried over-the-counter antacids and they were helping to relieve his symptoms for a few months, but they are not working well now. He denies dysphagia, odynophagia, or weight loss. You decide to treat him with a proton pump inhibitor at this visit, and he achieves good symptomatic relief with this therapy. What length of therapy is appropriate in this patient? A Two to four weeks B Eight to twelve weeks C Four to six months D One year E Continue indefinitely - ✔✔If a patient achieves good symptomatic relief with a course of an empiric, once-daily proton pump inhibitor, therapy may be discontinued after eight to twelve weeks. A 34-year-old male presents to the primary care office with a complaint of heartburn that has been present for three months. He has symptoms two to three times a week, which occurs about 30 minutes after eating. He has tried over-the-counter antacids and they were helping to relieve his symptoms for a few months, but they are not working well now. He denies dysphagia, odynophagia, or weight loss. You decide to treat him with a proton pump inhibitor at this visit, and he achieves good symptomatic relief with this therapy. What length of therapy is appropriate in this patient? A Two to four weeks B Eight to twelve weeks C Four to six months D One year E Continue indefinitely - ✔✔ A 63 year-old woman complains of fatigue, loss of appetite, a sore-red tongue, paresthesias of her feet and hands, and unsteadiness of her gait. Which of the following tests is be used to confirm the patient's suspected diagnosis? A Anti-intrinsic factor antibodies B Antiparietal cell antibodies C Coomb's test D Schilling Test E Serum folate levels - ✔✔Anti-intrinsic factor antibodies A diagnosis of pernicious anemia (PA) can be supported through the presence of anti-intrinsic factor antibodies. Antiparietal (B) cell antibodies aren't associated with the development of PA. The Coomb's test (C) is used in the evaluation of hemolytic anemias. Serum folate (D) levels are beneficial in evaluating macrocytic anemias, but will not establish a diagnosis of PA. The Schilling test (D) was once commonly used to diagnosis PA, but is no longer available due to lack of available radiolabeled human intrinsic factor. A 63 year-old woman complains of fatigue, loss of appetite, a sore-red tongue, paresthesias of her feet and hands, and unsteadiness of her gait. Which of the following tests is be used to confirm the patient's suspected diagnosis? A Anti-intrinsic factor antibodies B Antiparietal cell antibodies C Coomb's test D Schilling Test E Serum folate levels - ✔✔Anti-intrinsic factor antibodies A diagnosis of pernicious anemia (PA) can be supported through the presence of anti-intrinsic factor antibodies. Antiparietal (B) cell antibodies aren't associated with the development of PA. The Coomb's test (C) is used in the evaluation of hemolytic anemias. Serum folate (D) levels are beneficial in evaluating macrocytic anemias, but will not establish a diagnosis of PA. The Schilling test (D) was once commonly used to diagnosis PA, but is no longer available due to lack of available radiolabeled human intrinsic factor. A 63 year-old woman complains of fatigue, loss of appetite, a sore-red tongue, paresthesias of her feet and hands, and unsteadiness of her gait. Which of the following tests is be used to confirm the patient's suspected diagnosis? A Anti-intrinsic factor antibodies B Antiparietal cell antibodies C Coomb's test D Schilling Test E Serum folate levels - ✔✔Anti-intrinsic factor antibodies A diagnosis of pernicious anemia (PA) can be supported through the presence of anti-intrinsic factor antibodies. Antiparietal (B) cell antibodies aren't associated with the development of PA. The Coomb's test (C) is used in the evaluation of hemolytic anemias. Serum folate (D) levels are beneficial in evaluating macrocytic anemias, but will not establish a diagnosis of PA. The Schilling test (D) was once commonly used to diagnosis PA, but is no longer available due to lack of available radiolabeled human intrinsic factor. A 63 year-old woman complains of fatigue, loss of appetite, a sore-red tongue, paresthesias of her feet and hands, and unsteadiness of her gait. Which of the following tests is be used to confirm the patient's suspected diagnosis? A Anti-intrinsic factor antibodies B Antiparietal cell antibodies C Coomb's test D Schilling Test E Serum folate levels - ✔✔Anti-intrinsic factor antibodies A diagnosis of pernicious anemia (PA) can be supported through the presence of anti-intrinsic factor antibodies. Antiparietal (B) cell antibodies aren't associated with the development of PA. The Coomb's test (C) is used in the evaluation of hemolytic anemias. Serum folate (D) levels are beneficial in evaluating macrocytic anemias, but will not establish a diagnosis of PA. The Schilling test (D) was once commonly used to diagnosis PA, but is no longer available due to lack of available radiolabeled human intrinsic factor. - ✔✔A diagnosis of pernicious anemia (PA) can be supported through the presence of anti-intrinsic factor antibodies. Antiparietal (B) cell antibodies aren't associated with the development of PA. The Coomb's test (C) is used in the evaluation of hemolytic anemias. Serum folate (D) levels are beneficial in evaluating macrocytic anemias, but will not establish a diagnosis of PA. The Schilling test (D) was once commonly used to diagnosis PA, but is no longer available due to lack of available radiolabeled human intrinsic factor. A 28-year-old man presents for a barium enema; he has a 6-month history of abdominal cramping, pain, tenesmus, bloody diarrhea, and painful urgency. He also reports arthralgias, but denies a history of traveling abroad or the use of antibiotics. On physical exam, he has indurated and painful erythematous plaques and nodules on both shins. An abdominal exam reveals a mild tenderness in the lower left quadrant (LLQ). Stool exam is positive for occult blood and negative for ova and parasites. Lab analysis reveals mild leukocytosis, microcytic anemia, positive p-ANCA antibodies, and elevated ESR. What would be the most likely result of a barium enema? Answer Choices 1 Apple core appearance 2 Cobblestone appearance 3 Lead pipe appearance 4 String sign 5 Thumb printing - ✔✔lead pipe appearance on barium enema Lead pipe colon is the appearance on barium enema of a foreshortened, narrow colon with loss of redundancy and haustral markings. This sign is specific for ulcerative colitis (UC), an inflammatory disease of the rectum and colon. The peak incidence of UC is around 25 to 45 years. P-ANCA antibodies are associated with UC. Iron deficiency can lead to microcytic anemia. Common extra intestinal manifestations of UC are erythema nodosum (described as erythematous plaques and nodules on pretibial areas), arthritis, pyoderma gangrenosum, uveitis, episcleritis, and sclerosing cholangitis. Toxic megacolon is a complication of UC characterized by the dilation of the colon; there is a risk of perforation. There is an increased incidence of colon cancer (10%) after 10 years of this disease. The inflammation involves the mucosal tissues only, which are friable and show ulcerations on colonoscopy. There is also a characteristic continuous involvement and pseudopolyp appearance caused by mucosal regeneration. Sulfapyridine, mesalamine, corticosteroids, 6-mercaptopurine, and azathioprine are used for treatment. Colectomy is curative and is indicated for intractable disease, or in the case of dysplasia. An apple core appearance on barium enema is seen in patients with colon cancer. This sign refers to the shouldered margins of the stricture caused by the neoplasmic invasion. A cobblestone appearance and string sign are characteristic for Crohn's disease, another inflammatory disease of the digestive tract. The cobblestone appearance refers to the discontinuous areas of inflammation separated by healthy bowel. String sign, seen in the small bowel series, is the appearance caused by stricture areas. Thumb printing is a radiological sign seen in ischemic colitis, a disease most likely found in elderly patients with left lower abdominal pain and gross rectal bleeding. A 28-year-old man presents for a barium enema; he has a 6-month history of abdominal cramping, pain, tenesmus, bloody diarrhea, and painful urgency. He also reports arthralgias, but denies a history of traveling abroad or the use of antibiotics. On physical exam, he has indurated and painful erythematous plaques and nodules on both shins. An abdominal exam reveals a mild tenderness in the lower left quadrant (LLQ). Stool exam is positive for occult blood and negative for ova and parasites. Lab analysis reveals mild leukocytosis, microcytic anemia, positive p-ANCA antibodies, and elevated ESR. What would be the most likely result of a barium enema? Answer Choices 1 Apple core appearance 2 Cobblestone appearance 3 Lead pipe appearance 4 String sign 5 Thumb printing - ✔✔lead pipe appearance on barium enema Lead pipe colon is the appearance on barium enema of a foreshortened, narrow colon with loss of redundancy and haustral markings. This sign is specific for ulcerative colitis (UC), an inflammatory disease of the rectum and colon. The peak incidence of UC is around 25 to 45 years. P-ANCA antibodies are associated with UC. Iron deficiency can lead to microcytic anemia. Common extra intestinal manifestations of UC are erythema nodosum (described as erythematous plaques and nodules on pretibial areas), arthritis, pyoderma gangrenosum, uveitis, episcleritis, and sclerosing cholangitis. Toxic megacolon is a complication of UC characterized by the dilation of the colon; there is a risk of perforation. There is an increased incidence of colon cancer (10%) after 10 years of this disease. The inflammation involves the mucosal tissues only, which are friable and show ulcerations on colonoscopy. There is also a characteristic continuous involvement and pseudopolyp appearance caused by mucosal regeneration. Sulfapyridine, mesalamine, corticosteroids, 6-mercaptopurine, and azathioprine are used for treatment. Colectomy is curative and is indicated for intractable disease, or in the case of dysplasia. An apple core appearance on barium enema is seen in patients with colon cancer. This sign refers to the shouldered margins of the stricture caused by the neoplasmic invasion. A cobblestone appearance and string sign are characteristic for Crohn's disease, another inflammatory disease of the digestive tract. The cobblestone appearance refers to the discontinuous areas of inflammation separated by healthy bowel. String sign, seen in the small bowel series, is the appearance caused by stricture areas. Thumb printing is a radiological sign seen in ischemic colitis, a disease most likely found in elderly patients with left lower abdominal pain and gross rectal bleeding. A 28-year-old man presents for a barium enema; he has a 6-month history of abdominal cramping, pain, tenesmus, bloody diarrhea, and painful urgency. He also reports arthralgias, but denies a history of traveling abroad or the use of antibiotics. On physical exam, he has indurated and painful erythematous plaques and nodules on both shins. An abdominal exam reveals a mild tenderness in the lower left quadrant (LLQ). Stool exam is positive for occult blood and negative for ova and parasites. Lab analysis reveals mild leukocytosis, microcytic anemia, positive p-ANCA antibodies, and elevated ESR. What would be the most likely result of a barium enema? Answer Choices 1 Apple core appearance 2 Cobblestone appearance 3 Lead pipe appearance 4 String sign 5 Thumb printing - ✔✔lead pipe appearance on barium enema Lead pipe colon is the appearance on barium enema of a foreshortened, narrow colon with loss of redundancy and haustral markings. This sign is specific for ulcerative colitis (UC), an inflammatory disease of the rectum and colon. The peak incidence of UC is around 25 to 45 years. P-ANCA antibodies are associated with UC. Iron deficiency can lead to microcytic anemia. Common extra intestinal manifestations of UC are erythema nodosum (described as erythematous plaques and nodules on pretibial areas), arthritis, pyoderma gangrenosum, uveitis, episcleritis, and sclerosing cholangitis. Toxic megacolon is a complication of UC characterized by the dilation of the colon; there is a risk of perforation. There is an increased incidence of colon cancer (10%) after 10 years of this disease. The inflammation involves the mucosal tissues only, which are friable and show ulcerations on colonoscopy. There is also a characteristic continuous involvement and pseudopolyp appearance caused by mucosal regeneration. Sulfapyridine, mesalamine, corticosteroids, 6-mercaptopurine, and azathioprine are used for treatment. Colectomy is curative and is indicated for intractable disease, or in the case of dysplasia. An apple core appearance on barium enema is seen in patients with colon cancer. This sign refers to the shouldered margins of the stricture caused by the neoplasmic invasion. A cobblestone appearance and string sign are characteristic for Crohn's disease, another inflammatory disease of the digestive tract. The cobblestone appearance refers to the discontinuous areas of inflammation separated by healthy bowel. String sign, seen in the small bowel series, is the appearance caused by stricture areas. Thumb printing is a radiological sign seen in ischemic colitis, a disease most likely found in elderly patients with left lower abdominal pain and gross rectal bleeding. A 15-year-old boy was healthy until 3 months ago. He has been having episodes of crampy abdominal pain and explosive liquid stools 2-3 times a day. He has had intermittent fevers and has noted blood in the stool. He has a documented 15-lb weight loss. Question What is the best way to start treatment? Answer Choices 1 Diphenoxylate hydrochloride with atropine sulfate 2 Eliminate lactose-containing foods 3 Loperamide HCL 4 Eliminate all high fiber foods 5 Sulfasalazine - ✔✔Sulfasalazine This patient has ulcerative colitis. Therefore, it would be most appropriate to start treatment with sulfasalazine at 50-75 mg/kg/day in 2-4 divided doses. Sulfasalazine is a sulfa drug that has important anti-inflammatory properties because it inhibits the synthesis of mediators of the inflammatory response. In some patients, sulfasalazine is poorly tolerated, in which case mesalamine (50-100 mg/kg/day) and balsalazide (110-175 mg/kg/day) are preferable treatments. Diphenoxylate hydrochloride with atropine sulfate at 1-2 mL t.i.d. and loperamide HCl at 0.04-0.08 mg/kg/day in 2-4 divided doses are contraindicated because they would suppress the symptoms of ulcerative colitis without amelioration of the underlying cause of the condition. Elimination of lactose-containing food would have no effect because the patient is lactose-tolerant. In addition, elimination of dietary fiber would not have any effect. A 15-year-old girl presents with a 1-year history of intermittent abdominal pain with nausea and occasional bloody diarrhea. She denies fever or weight loss, as well as any travel history. Past medical history is significant only for migraines. She takes a multivitamin. Her vital signs are within normal limits. She has mild diffuse abdominal tenderness to palpation and guaiac-positive stool. Her exam is otherwise normal. Her hemoglobin is 9.7, hematocrit is 28%, and her WBC is 12,000/uL. Question What finding would indicate a possible cure if a colectomy is performed? Answer Choices 1 Skip lesions, transmural involvement on colonoscopy 2 Crypt abscesses, superficial mucosal involvement on colonoscopy 3 Ileal disease 4 Fistulas, fissures 5 Mouth ulcers - ✔✔Crypt abscesses, superficial mucosal involvement on colonoscopy Crypt abscesses and superficial mucosal involvement are colonoscopy findings associated with ulcerative colitis. A colectomy is performed for severe and intractable disease. A colectomy can be a relative cure since the disease is limited to the colon. Skip lesions, transmural involvement, fistulas, and fissures are all associated with colonoscopy findings in Crohn's disease. Crohn's disease can also affect any structure of the alimentary tract from the mouth to anus, causing ileal disease and mouth ulcers. A 15-year-old girl presents with a 1-year history of intermittent abdominal pain with nausea and occasional bloody diarrhea. She denies fever or weight loss, as well as any travel history. Past medical history is significant only for migraines. She takes a multivitamin. Her vital signs are within normal limits. She has mild diffuse abdominal tenderness to palpation and guaiac-positive stool. Her exam is otherwise normal. Her hemoglobin is 9.7, hematocrit is 28%, and her WBC is 12,000/uL. Question What finding would indicate a possible cure if a colectomy is performed? Answer Choices 1 Skip lesions, transmural involvement on colonoscopy 2 Crypt abscesses, superficial mucosal involvement on colonoscopy 3 Ileal disease 4 Fistulas, fissures 5 Mouth ulcers - ✔✔Crypt abscesses, superficial mucosal involvement on colonoscopy Crypt abscesses and superficial mucosal involvement are colonoscopy findings associated with ulcerative colitis. A colectomy is performed for severe and intractable disease. A colectomy can be a relative cure since the disease is limited to the colon. Skip lesions, transmural involvement, fistulas, and fissures are all associated with colonoscopy findings in Crohn's disease. Crohn's disease can also affect any structure of the alimentary tract from the mouth to anus, causing ileal disease and mouth ulcers. A 15-year-old girl presents with a 1-year history of intermittent abdominal pain with nausea and occasional bloody diarrhea. She denies fever or weight loss, as well as any travel history. Past medical history is significant only for migraines. She takes a multivitamin. Her vital signs are within normal limits. She has mild diffuse abdominal tenderness to palpation and guaiac-positive stool. Her exam is otherwise normal. Her hemoglobin is 9.7, hematocrit is 28%, and her WBC is 12,000/uL. Question What finding would indicate a possible cure if a colectomy is performed? Answer Choices 1 Skip lesions, transmural involvement on colonoscopy 2 Crypt abscesses, superficial mucosal involvement on colonoscopy 3 Ileal disease 4 Fistulas, fissures 5 Mouth ulcers - ✔✔Crypt abscesses, superficial mucosal involvement on colonoscopy Crypt abscesses and superficial mucosal involvement are colonoscopy findings associated with ulcerative colitis. A colectomy is performed for severe and intractable disease. A colectomy can be a relative cure since the disease is limited to the colon. Skip lesions, transmural involvement, fistulas, and fissures are all associated with colonoscopy findings in Crohn's disease. Crohn's disease can also affect any structure of the alimentary tract from the mouth to anus, causing ileal disease and mouth ulcers. A 15-year-old girl presents with a 1-year history of intermittent abdominal pain with nausea and occasional bloody diarrhea. She denies fever or weight loss, as well as any travel history. Past medical history is significant only for migraines. She takes a multivitamin. Her vital signs are within normal limits. She has mild diffuse abdominal tenderness to palpation and guaiac-positive stool. Her exam is otherwise normal. Her hemoglobin is 9.7, hematocrit is 28%, and her WBC is 12,000/uL. Question What finding would indicate a possible cure if a colectomy is performed? Answer Choices 1 Skip lesions, transmural involvement on colonoscopy 2 Crypt abscesses, superficial mucosal involvement on colonoscopy 3 Ileal disease 4 Fistulas, fissures 5 Mouth ulcers - ✔✔Crypt abscesses, superficial mucosal involvement on colonoscopy Crypt abscesses and superficial mucosal involvement are colonoscopy findings associated with ulcerative colitis. A colectomy is performed for severe and intractable disease. A colectomy can be a relative cure since the disease is limited to the colon. Skip lesions, transmural involvement, fistulas, and fissures are all associated with colonoscopy findings in Crohn's disease. Crohn's disease can also affect any structure of the alimentary tract from the mouth to anus, causing ileal disease and mouth ulcers. A 15-year-old girl presents with a 1-year history of intermittent abdominal pain with nausea and occasional bloody diarrhea. She denies fever or weight loss, as well as any travel history. Past medical history is significant only for migraines. She takes a multivitamin. Her vital signs are within normal limits. She has mild diffuse abdominal tenderness to palpation and guaiac-positive stool. Her exam is otherwise normal. Her hemoglobin is 9.7, hematocrit is 28%, and her WBC is 12,000/uL. Question What finding would indicate a possible cure if a colectomy is performed? Answer Choices 1 Skip lesions, transmural involvement on colonoscopy 2 Crypt abscesses, superficial mucosal involvement on colonoscopy 3 Ileal disease 4 Fistulas, fissures 5 Mouth ulcers - ✔✔Crypt abscesses, superficial mucosal involvement on colonoscopy Crypt abscesses and superficial mucosal involvement are colonoscopy findings associated with ulcerative colitis. A colectomy is performed for severe and intractable disease. A colectomy can be a relative cure since the disease is limited to the colon. Skip lesions, transmural involvement, fistulas, and fissures are all associated with colonoscopy findings in Crohn's disease. Crohn's disease can also affect any structure of the alimentary tract from the mouth to anus, causing ileal disease and mouth ulcers. A 15-year-old girl presents with a 1-year history of intermittent abdominal pain with nausea and occasional bloody diarrhea. She denies fever or weight loss, as well as any travel history. Past medical history is significant only for migraines. She takes a multivitamin. Her vital signs are within normal limits. She has mild diffuse abdominal tenderness to palpation and guaiac-positive stool. Her exam is otherwise normal. Her hemoglobin is 9.7, hematocrit is 28%, and her WBC is 12,000/uL. Question What finding would indicate a possible cure if a colectomy is performed? Answer Choices 1 Skip lesions, transmural involvement on colonoscopy 2 Crypt abscesses, superficial mucosal involvement on colonoscopy 3 Ileal disease 4 Fistulas, fissures 5 Mouth ulcers - ✔✔Crypt abscesses, superficial mucosal involvement on colonoscopy Crypt abscesses and superficial mucosal involvement are colonoscopy findings associated with ulcerative colitis. A colectomy is performed for severe and intractable disease. A colectomy can be a relative cure since the disease is limited to the colon. Skip lesions, transmural involvement, fistulas, and fissures are all associated with colonoscopy findings in Crohn's disease. Crohn's disease can also affect any structure of the alimentary tract from the mouth to anus, causing ileal disease and mouth ulcers. A 15-year-old girl presents with a 1-year history of intermittent abdominal pain with nausea and occasional bloody diarrhea. She denies fever or weight loss, as well as any travel history. Past medical history is significant only for migraines. She takes a multivitamin. Her vital signs are within normal limits. She has mild diffuse abdominal tenderness to palpation and guaiac-positive stool. Her exam is otherwise normal. Her hemoglobin is 9.7, hematocrit is 28%, and her WBC is 12,000/uL. Question What finding would indicate a possible cure if a colectomy is performed? Answer Choices 1 Skip lesions, transmural involvement on colonoscopy 2 Crypt abscesses, superficial mucosal involvement on colonoscopy 3 Ileal disease 4 Fistulas, fissures 5 Mouth ulcers - ✔✔Crypt abscesses, superficial mucosal involvement on colonoscopy Crypt abscesses and superficial mucosal involvement are colonoscopy findings associated with ulcerative colitis. A colectomy is performed for severe and intractable disease. A colectomy can be a relative cure since the disease is limited to the colon. Skip lesions, transmural involvement, fistulas, and fissures are all associated with colonoscopy findings in Crohn's disease. Crohn's disease can also affect any structure of the alimentary tract from the mouth to anus, causing ileal disease and mouth ulcers. A 15-year-old girl presents with a 1-year history of intermittent abdominal pain with nausea and occasional bloody diarrhea. She denies fever or weight loss, as well as any travel history. Past medical history is significant only for migraines. She takes a multivitamin. Her vital signs are within normal limits. She has mild diffuse abdominal tenderness to palpation and guaiac-positive stool. Her exam is otherwise normal. Her hemoglobin is 9.7, hematocrit is 28%, and her WBC is 12,000/uL. Question What finding would indicate a possible cure if a colectomy is performed? Answer Choices 1 Skip lesions, transmural involvement on colonoscopy 2 Crypt abscesses, superficial mucosal involvement on colonoscopy 3 Ileal disease 4 Fistulas, fissures 5 Mouth ulcers - ✔✔Crypt abscesses, superficial mucosal involvement on colonoscopy Crypt abscesses and superficial mucosal involvement are colonoscopy findings associated with ulcerative colitis. A colectomy is performed for severe and intractable disease. A colectomy can be a relative cure since the disease is limited to the colon. Skip lesions, transmural involvement, fistulas, and fissures are all associated with colonoscopy findings in Crohn's disease. Crohn's disease can also affect any structure of the alimentary tract from the mouth to anus, causing ileal disease and mouth ulcers. A 15-year-old girl presents with a 1-year history of intermittent abdominal pain with nausea and occasional bloody diarrhea. She denies fever or weight loss, as well as any travel history. Past medical history is significant only for migraines. She takes a multivitamin. Her vital signs are within normal limits. She has mild diffuse abdominal tenderness to palpation and guaiac-positive stool. Her exam is otherwise normal. Her hemoglobin is 9.7, hematocrit is 28%, and her WBC is 12,000/uL. Question What finding would indicate a possible cure if a colectomy is performed? Answer Choices 1 Skip lesions, transmural involvement on colonoscopy 2 Crypt abscesses, superficial mucosal involvement on colonoscopy 3 Ileal disease 4 Fistulas, fissures 5 Mouth ulcers - ✔✔Crypt abscesses, superficial mucosal involvement on colonoscopy Crypt abscesses and superficial mucosal involvement are colonoscopy findings associated with ulcerative colitis. A colectomy is performed for severe and intractable disease. A colectomy can be a relative cure since the disease is limited to the colon. Skip lesions, transmural involvement, fistulas, and fissures are all associated with colonoscopy findings in Crohn's disease. Crohn's disease can also affect any structure of the alimentary tract from the mouth to anus, causing ileal disease and mouth ulcers. A 15-year-old girl presents with a 1-year history of intermittent abdominal pain with nausea and occasional bloody diarrhea. She denies fever or weight loss, as well as any travel history. Past medical history is significant only for migraines. She takes a multivitamin. Her vital signs are within normal limits. She has mild diffuse abdominal tenderness to palpation and guaiac-positive stool. Her exam is otherwise normal. Her hemoglobin is 9.7, hematocrit is 28%, and her WBC is 12,000/uL. Question What finding would indicate a possible cure if a colectomy is performed? Answer Choices 1 Skip lesions, transmural involvement on colonoscopy 2 Crypt abscesses, superficial mucosal involvement on colonoscopy 3 Ileal disease 4 Fistulas, fissures 5 Mouth ulcers - ✔✔Crypt abscesses, superficial mucosal involvement on colonoscopy Crypt abscesses and superficial mucosal involvement are colonoscopy findings associated with ulcerative colitis. A colectomy is performed for severe and intractable disease. A colectomy can be a relative cure since the disease is limited to the colon. Skip lesions, transmural involvement, fistulas, and fissures are all associated with colonoscopy findings in Crohn's disease. Crohn's disease can also affect any structure of the alimentary tract from the mouth to anus, causing ileal disease and mouth ulcers. A 15-year-old girl presents with a 1-year history of intermittent abdominal pain with nausea and occasional bloody diarrhea. She denies fever or weight loss, as well as any travel history. Past medical history is significant only for migraines. She takes a multivitamin. Her vital signs are within normal limits. She has mild diffuse abdominal tenderness to palpation and guaiac-positive stool. Her exam is otherwise normal. Her hemoglobin is 9.7, hematocrit is 28%, and her WBC is 12,000/uL. Question What finding would indicate a possible cure if a colectomy is performed? Answer Choices 1 Skip lesions, transmural involvement on colonoscopy 2 Crypt abscesses, superficial mucosal involvement on colonoscopy 3 Ileal disease 4 Fistulas, fissures 5 Mouth ulcers - ✔✔ A 13-year-old boy presents with a long history of intermittent jaundice without other signs and symptoms. Yesterday he took several paracetamol tablets, and the headache and jaundice appeared again. His parents have a consanguineous marriage, and nobody in the family has similar symptoms. His physical examination today is within normal limits, except for mild scleral icterus. The initial laboratory examinations show: RBC 4.5mill/mm3; WBC 6000/mm3; total bilirubin 2.2 mg/dl; indirect bilirubin 2.0 mg/dl; direct bili 0.2 mg/dl; liver enzymes, serum copper, and all other parameters are normal; hepatitis B (-); and the full set of his autoimmune markers (-) and Coombs test (-). Blood smear is normal. Ultrasonography of liver, gallbladder, pancreas, spleen, and for both kidneys are normal. His urine appears very yellow. Question What is the most likely the diagnosis? Answer Choices 1 Gilbert's syndrome 2 Intravascular hemolysis 3 Chronic viral hepatitis 4 Crigler-Najjar syndrome 5 Budd-Chiari syndrome - ✔✔Gilbert's disease Your patient most probably has Gilbert's syndrome (GS), also known as Gilbert-Meulengracht syndrome. It is a relatively common genetic disease found in up to 5% - 10% of the population and generally does not need special treatment. Inherited non-haemolytic hyperbilirubinemic conditions include Dubin-Johnson, Rotor, and GB syndromes, and all are important differential diagnoses indicating benign disease that requires no immediate treatment. GB can be diagnosed by clinical presentation, biochemistry, and genotyping, and is significant because of the presence of the disposition towards drug-associated toxicity. A major characteristic is jaundice, caused by elevated levels of unconjugated bilirubin in the bloodstream. The cause of this hyperbilirubinemia is the reduced activity of the enzyme glucuronyltransferase, which conjugates both bilirubin and some lipophilic molecules, including drugs. Intravascular hemolysis, with resulting hemoglobinemia, hemoglobinuria, and bilirubinemia, will show fragments of the red blood cells ("schistocytes") and sometimes spherocytes in peripheral blood smear, reticulocytosis, elevated unconjugated bilirubin that may lead to jaundice, elevated lactate dehydrogenase (LDH) in the blood, and decreased haptoglobin levels. If the direct Coombs test is positive, hemolysis is caused by an immune process. Hemosiderin in the urine indicates chronic intravascular hemolysis. There is also urobilinogen in the urine. Viral hepatitis with jaundice will have elevated liver function tests (AST and ALT elevated out of proportion to alkaline phosphatase, usually with hyperbilirubinemia), and viral serologic testing will be positive. Crigler-Najjar syndrome is a rare inherited form of non-hemolytic jaundice, which results in high levels of unconjugated bilirubin and often leads to brain damage in infancy. Budd-Chiari syndrome is caused by occlusion of the hepatic veins. It presents with the classical triad of abdominal pain, ascites, and hepatomegaly. The syndrome can be fulminant, acute, chronic, or asymptomatic. A 52-year-old Caucasian man presents with easy fatigability, poor concentration, and "stronger than usual" effort-related palpitations. He denies any constipation or weight loss. His past medical history includes type II diabetes, which was diagnosed 10 years ago; it is well controlled with diet and metformin, and there are no complications. Both his parents are in good health; his sister had a malignant breast nodule removed 2 years ago, but she showed no signs of metastatic disease. His physical examination shows mild obesity (BMI 32 kg/m2), but it is otherwise normal. A laboratory workup shows glucose 91 mg/dL, Na+ 138 mEq/L, K+ 4.9 mEq/L, Cl- 100 mEq/L, hemoglobin 11 g/dL, hematocrit 32%, mean corpuscular volume(MCV) 82 fl, mean corpuscular hemoglobin concentration (MCHC) 24 pg/mL, leukocytes 7,400 /mm3 w/normal differential. Platelets 310,000 /mm3, ferritin 29 ng/L (25-300 ng/mL), and a total iron binding capacity of 450 ng/dL. Question What is the most appropriate next step in diagnosis? Answer Choices 1 Fecal occult blood testing (FOBT) 2 Full colonoscopy 3 Bone marrow biopsy 4 Double-contrast barium enema 5 Capsule endoscopy - ✔✔The correct response is fecal occult blood testing. The pattern of low mean corpuscular volume and hemoglobin concentration, low ferritin, and elevated total iron-binding capacity (TIBC) is diagnostic of iron deficiency. Lack of iron inhibits hemoglobin synthesis, thereby reducing the volume and hemoglobin concentration of erythrocytes. Some patients have thrombocytosis. The mechanism is unclear, but hypotheses include stimulation by high circulating erythropoietin concentrations or circulating cytokines. Besides asthenia and pallor, other symptoms include pagophagia (craving ice or cold foods), pica (clay eating), and leg cramps; physical examination may reveal esophageal webbing, koilonychia (spoon-shaped nails), glossitis, angular stomatitis, and gastric atrophy. New-onset anemia with hypochromia and microcytosis in an adult should be considered secondary to gastrointestinal bleeding until proved otherwise. The most common etiologies are diverticular disease, angiodysplasia, colitis (including from NSAIDs), and colon cancer, particularly left-sided colon cancer. Testing for occult blood in a stool specimen is the initial diagnostic method of choice. If the fecal occult blood test is positive, the patient should undergo upper endoscopy and colonoscopy examinations. Colonoscopy is a useful exam for identifying sources of occult gastrointestinal bleeding. It is more sensitive than barium enema or CT scans for early lesions and permits biopsy when indicated (e.g., lesions suspicious for malignancy or colitis). However, none of these are cost-effective screening examinations. Bone marrow biopsy can evaluate bone marrow iron stores and reveal erythroid hyperplasia. However, it has been practically abandoned in the workup of iron-deficiency anemia, as there is considerable interobserver variability and assays for iron, ferritin, and TIBC permit evaluation of the body's iron stores less invasively. Capsule endoscopy is the most sensitive method for diagnosing small-bowel bleeding. However, small-bowel bleeding is much less common than upper (proximal to the angle of Treitz) or colonic bleeding. Therefore, it should only be undertaken after colonoscopy and upper endoscopy are negative. A 35-year-old Hispanic man is too embarrassed to explain his chief symptom. You enter the room and coerce him to give you the reason he has come for medical treatment. He admits to having a sore that will not heal around his rectum and anal area. This lesion has been draining pus consistently for the last week. The patient further explains that he has only had intermittent pain with this lesion (4/10 on a 0 - 10 pain scale) that is made slightly worse when he has a bowel movement (ranking it a 6/10). More recently, especially in the last few days, he has noted that simply sitting increases the pain. The patient also admits to intermittent periods of itching. He denies fever, diarrhea, or ever being diagnosed with inflammatory bowel disease. Physical examination of the anorectal area reveals excoriated and inflamed perianal skin with a palpated induration. Question What is this patient's most likely diagnosis? Answer Choices 1 Anal fistula 2 Crohn's disease 3 Anal fissure 4 Anal cancer 5 Internal hemorrhoid - ✔✔The patient in this scenario is experiencing signs and symptoms from an anorectal fistula. Anorectal fistulas are commonly a chronic manifestation of an acute perirectal process that eventually comes from an anorectal abscess. Many fistulas originate from an infected anal crypt gland. Patients with anorectal fistulas usually will present with a "non-healing" area (an anorectal abscess that is draining) or may describe chronic purulent drainage and a pustule-like lesion on the perianal or buttock area. They will only have intermittent pruritus or rectal pain; pain is increased during defecation, sitting, or standing. Physical examination reveals excoriated and inflamed perianal skin. An external opening of the fistula may be visualized. If no opening is seen, palpation will reveal induration just below the skin. Anal fissure is not the most likely diagnosis in this case scenario. Anal fissures most often affect infants as well as middle-aged individuals. The majority of fissures are considered primary and caused by local trauma such as passage of hard stool, prolonged diarrhea, vaginal delivery, or anal sex. Presentation of anal fissures is a tearing pain accompanying bowel movements as well as bright red rectal bleeding that is limited to a small amount noted on the toilet paper or surface of the stool. The patient described all of these components. Patients will also complain of perianal pruritus or irritation, which he also admits to experiencing. Patients with Crohn's disease often have extremely variable signs and symptoms and possibly will have these for many years prior to diagnosis. Fatigue, prolonged diarrhea and abdominal pain, weight loss, and fever with or without gross bleeding are considered hallmarks of Crohn's disease. Signs and symptoms of anal carcinoma include rectal bleeding (occurring in 45% of patients), anorectal pain or sensation of a rectal mass (30%), or even no issues (20%). Very often issues are confused with hemorrhoids and therefore patients may delay evaluation. Over 80% of patients developing anal cancer have detected human papilloma virus. Internal hemorrhoids create principle issues of bleeding, prolapse, and mucoid discharge.Bleeding may range from bright red blood that is seen as streaks on the toilet paper to actual dripping of it into the toilet bowl after a bowel movement. Over time, hemorrhoids potentially will develop into a prolapsed state achieve a point in time when patients will note a sense of fullness or discomfort along with mucoid drainage that results in further irritation and soiling of underclothes. A 46-year-old woman presents with a 2-month history of heartburn, epigastric discomfort, nausea, and occasional vomiting. She has a history of hyperlipidemia, controlled with diet and exercise, as well as asthma, for which she takes inhalers as needed. She takes no other medications, including over-the-counter analgesics. Family history is noncontributory. On exam she is afebrile, BP120/70 mm Hg, pulse 74/min, and SPO2 92%. Lungs are clear, and she has minimal epigastric tenderness. Otherwise, physical examination is unremarkable. She is advised by her physician to take lansoprazole once daily, which provides only partial relief. Endoscopy is then recommended, which shows a duodenal ulcer. Biopsy reveals infection with Helicobacter pylori. What would be the recommended regimen at this time? Answer Choices 1 Lansoprazole, amoxicillin, and metronidazole twice daily for 2 weeks 2 PPI (proton pump inhibitor), amoxicillin, and clarithromycin twice daily for 2 weeks 3 Omeprazole, tetracycline, and clarithromycin twice daily for 2 weeks 4 Bismuth, metronidazole, and lansoprazole twice daily for 2 weeks 5 Bismuth, metronidazole, and tetracycline 4 times daily for 2 weeks - ✔✔PPI (proton pump inhibitor), amoxicillin, and clarithromycin twice daily for 2 weeks There are several regimens recommended for H. pylori infection, which is an important cause of peptic ulcer disease and should be treated if found associated with symptoms. The choice of the regimen depends on considerations such as cost, side effects, and ease of administration. Allergy to one of the medications, as well as intolerance, should also be taken into account. Any proton pump inhibitor (PPI) with amoxicillin 1000 mg twice daily and clarithromycin 500 mg twice daily for 2 weeks or PPI with metronidazole 500mg twice daily and clarithromycin 500mg twice daily for 2 weeks are recommended. These are the triple drug therapies available. The other regimens suggested are bismuth, metronidazole, and tetracycline 4 times daily for 2 weeks along with PPI twice daily for 2 weeks or H2 receptor antagonist twice daily for 4 weeks (quadruple drug therapy). Dual therapy with a proton pump inhibitor and an antibiotic (amoxicillin or clarithromycin) is not recommended as primary therapy, since eradication rates are much lower than the above regimens. The most common side effect is a metallic taste in the mouth due to clarithromycin or metronidazole. Amoxicillin can cause diarrhea or a rash. Clarithromycin can also cause nausea, vomiting, abdominal pain, and (rarely) QT prolongation. Metronidazole can cause peripheral neuropathy, seizures, and a disulfiram-like reaction when taken with alcohol. Tetracycline is teratogenic and causes photosensitivity. Mandatory newborn screening has been conducted on an infant who was born 36 hours ago. The abnormality found in the screening has resulted in counseling on initiating lifelong modification to the newborn's diet. It is explained to the mother this action must be taken in order to help decrease the incidence of severe intellectual disability, hyperactivity, and even seizures. Question Considering the most likely diagnosis of this patient, which of the following must be limited in order to prevent these consequences? Answer Choices 1 Tyrosine 2 Fructose 3 Phenylalanine 4 Thyroid hormone 5 Leucine - ✔✔ A 45-year-old chronic alcoholic presents with history of massive hematemesis. This hematemesis followed a bout of prolonged vomiting. Patient has been a known alcoholic for 20 years. On examination, he has a pulse rate of 100/min and a BP of 90/70 mm Hg with cold extremities. These findings are typical for what condition? Answer Choices 1 Hiatus hernia 2 Esophageal pulsion diverticulum 3 Barrett's esophagus 4 Esophageal squamous cell carcinoma 5 Esophageal laceration - ✔✔Esophageal laceration Esophageal laceration extends to the submucosal veins that bleed profusely, also known as Mallory-Weiss syndrome. Hiatus hernia may cause a reflux along with inflammation and possible ulceration, but bleeding is not typically massive. Diverticula of the esophagus usually do not bleed. Barrett's esophagus (metaplasia with gastric mucosa) is associated with reflux and inflammation and possible ulceration, but bleeding is not usually massive. Esophageal squamous cell carcinoma will cause ulceration, but massive bleeding is not common. A 60-year-old man with a past medical history of COPD, hypertension, peptic ulcer disease, and hyperlipidemia is being evaluated for a 1-hour history of severe pain in the mid-abdominal region. His history includes abdominal surgery 6 months ago for a small intestinal obstruction, and his pain emanates from his incisional site. He also states that he has shaking chills, nausea, and vomiting, but denies chest pain, shortness of breath, wheezing, or abnormal bowel habits. Upon physical exam, he is found to be hypotensive, tachycardic, diaphoretic, and in acute painful distress. The abdomen is obese; bowel sounds are hypoactive; and rebound tenderness is noted. There is an exquisitely tender 2.0 cm ventral hernia located inferior to the umbilicus that is indurated, tender to palpation, and is not reducible. Auscultation of the mass reveals the presence of bowel sounds. Question What is the most appropriate therapeutic intervention for this patient at this time? Answer Choices 1 Broad spectrum antibiotics 2 Maintain a solid diet 3 Apply hot packs to the hernia 4 Oral motrin for pain 5 Attempt hernia reduction - ✔✔Broad spectrum antibiotics This patient's presentation is significant of a strangulated hernia. Manifestations of hernia strangulation, a medical emergency, include an exquisitely tender mass associated with systemic signs and symptoms, such as intestinal obstruction, toxic appearance, peritonitis, or meeting sepsis criteria. General surgery should be consulted immediately.Additional interventions include the administering broad-spectrum IV antibiotics, such as cefoxitin, providing fluid resuscitation and adequate narcotic analgesia, and obtaining preoperative laboratory studies. If there is any concern for strangulation, do not attempt hernia reduction. The reintroduction of ischemic, necrotic bowel back into the peritoneal cavity can result in subsequent perforation and sepsis. Bedside ED US, using a linear high frequency probe with color or power Doppler of the hernia sac can be useful in these borderline cases to establish the presence or absence of blood flow. What statement is true concerning balloon tamponade of esophageal varices? Answer Choices 1 An excellent measure for long term control of esophageal varices 2 Need not be preceded by upper endoscopy in a patient with a prior history of bleeding 3 Should be preceded by endotracheal intubation to protect the airway from aspiration 4 Position confirmation can be made by auscultation of the abdomen 5 Is associated with minimal morbidity and mortality - ✔✔Should be preceded by endotracheal intubation to protect the airway from aspiration The use of balloon tamponade, most commonly with the Sengstaken-Blakemore tube, for esophageal varices is temporarily effective in controlling bleeding in 80-90% of patients. However, rebleeding will occur in 60% of patients and therefore is not effective for long term control. This is not a benign procedure, there is a significant risk of aspiration and esophageal or gastric perforation. Therefore, elective endotracheal intubation prior to placement of the tube is preferential. Careful placement of the tube with meticulous monitoring of balloon pressures is essential to minimize the risks of perforation. Radiologic confirmation of the tube position should always precede inflation of the balloons and frequent radiologic studies should be performed to assure maintenance of this position. In a patient with a history of esophageal varices with a new upper gastrointestinal bleed, 25% of the time it will be from a source other than the varices. Therefore, if the patient is not having exsanguinating hemorrhage, it is useful to perform upper endoscopy to confirm the source of bleeding and possibly treat the varices with sclerotherapy, which can control the bleeding in 90-95% of cases. A 60-year-old man with a past medical history of COPD, hypertension, peptic ulcer disease, and hyperlipidemia is being evaluated for a 1-hour history of severe pain in the mid-abdominal region. His history includes abdominal surgery 6 months ago for a small intestinal obstruction, and his pain emanates from his incisional site. He also states that he has shaking chills, nausea, and vomiting, but denies chest pain, shortness of breath, wheezing, or abnormal bowel habits. Upon physical exam, he is found to be hypotensive, tachycardic, diaphoretic, and in acute painful distress. The abdomen is obese; bowel sounds are hypoactive; and rebound tenderness is noted. There is an exquisitely tender 2.0 cm ventral hernia located inferior to the umbilicus that is indurated, tender to palpation, and is not reducible. Auscultation of the mass reveals the presence of bowel sounds. Question What is the most appropriate therapeutic intervention for this patient at this time? Answer Choices 1 Broad spectrum antibiotics 2 Maintain a solid diet 3 Apply hot packs to the hernia 4 Oral motrin for pain 5 Attempt hernia reduction - ✔✔Broad spectrum antibiotics This patient's presentation is significant of a strangulated hernia. Manifestations of hernia strangulation, a medical emergency, include an exquisitely tender mass associated with systemic signs and symptoms, such as intestinal obstruction, toxic appearance, peritonitis, or meeting sepsis criteria. General surgery should be consulted immediately.Additional interventions include the administering broad-spectrum IV antibiotics, such as cefoxitin, providing fluid resuscitation and adequate narcotic analgesia, and obtaining preoperative laboratory studies. If there is any concern for strangulation, do not attempt hernia reduction. The reintroduction of ischemic, necrotic bowel back into the peritoneal cavity can result in subsequent perforation and sepsis. Bedside ED US, using a linear high frequency probe with color or power Doppler of the hernia sac can be useful in these borderline cases to establish the presence or absence of blood flow. A 56-year-old woman presents with heartburn and dyspepsia. She was diagnosed with osteoarthritis 4 years prior to presentation, and for the past 18 months, she has been managing pain with naproxen. An upper gastrointestinal endoscopy shows several areas of gastric irritation and a single ulcer about 3 mm in diameter. The gastroenterologist suggests that the patient be tested for Helicobacter pylori infection. What is the gold standard for diagnosis of this infection? Answer Choices 1 Culture 2 Histology 3 Antigen test 4 Western blot serology 5 Urease breath test - ✔✔Histology The long term use of non-steroidal antiinflammatory drugs (NSAIDS) in treatment of arthropathies carries about a 4-fold increase in the relative risk of duodenal or gastric ulcers. This risk is multifactorial; 1 risk factor for development of gastric ulcers in patients taking NSAIDS is infection with Helicobacter pylori. Histologic examination of gastric biopsies obtained during endoscopy is the gold standard for diagnosis of this infection based on sensitivity and specificity (refer to the table). This approach has the added benefit of allowing simultaneous assessment of the pathology of the lesion, which can rule out neoplasms. Culture of H. pylori is difficult and requires considerable microbiologic expertise. However, indicators in the cultivation medium can detect urease. Invasive procedures can be preceded by the urease breath test as a screening for the presence of the organism. Helicobacter serology by ELISA is limited to screening and detection of a past or present infection. An antigen test is also available that detects H. pylori protein in the stool. A 45-year-old chronic alcoholic presents with history of massive hematemesis. This hematemesis followed a bout of prolonged vomiting. Patient has been a known alcoholic for 20 years. On examination, he has a pulse rate of 100/min and a BP of 90/70 mm Hg with cold extremities. These findings are typical for what condition? Answer Choices 1 Hiatus hernia 2 Esophageal pulsion diverticulum 3 Barrett's esophagus 4 Esophageal squamous cell carcinoma 5 Esophageal laceration - ✔✔Esophageal laceration Esophageal laceration extends to the submucosal veins that bleed profusely, also known as Mallory-Weiss syndrome. Hiatus hernia may cause a reflux along with inflammation and possible ulceration, but bleeding is not typically massive. Diverticula of the esophagus usually do not bleed. Barrett's esophagus (metaplasia with gastric mucosa) is associated with reflux and inflammation and possible ulceration, but bleeding is not usually massive. Esophageal squamous cell carcinoma will cause ulceration, but massive bleeding is not common. A 45-year-old man is admitted to the medical floor of a hospital with increasing jaundice, swollen legs, and episodes of disorientation for the last several weeks. His abdomen is distended and he looks ill. According to the family, he is an alcoholic and has recently been diagnosed with cirrhosis of liver. On examination, he has a temperature of 99 F, BP 100/72 mm Hg, and his pulse is 86/minute. Sclerae are icteric, and he has 2+ pitting pedal edema. Lungs are clear, and heart sounds are normal. Abdomen is distended with moderate ascites, caput medusae, and no tenderness. Liver and spleen are not palpable. He is awake but drowsy and oriented to person and place but not to time. He does have a fine tremor in his hands. Labs show WBC 8000/uL, platelets 100,000/uL, Hb 12g%, AST 76 U/L, ALT 56 U/L, AP 62 U/L, and ammonia is 124 mg/dL. Question At this time which of the following is most important to avoid full decompensation into hepatic encephalopathy? Answer Choices 1 Increase dietary protein 2 Prevent constipation 3 Sedate the patient to avoid injury from disorientation 4 Start IV antibiotics empirically anticipating infection 5 Add thiamine and folic acid for nutritional support - ✔✔Prevent Constipation Explanation This patient is already in the initial phases of hepatic encephalopathy due to alcoholic liver disease as evidenced by the disorientation and tremors, also known as asterixis. The primary cause of hepatic encephalopathy is unclear. Metabolic abnormalities due to liver dysfunction, resulting in a spectrum of neuropsychiatric signs and symptoms, are seen. High levels of ammonia are found in the blood. Constipation causes increased ammonia production and absorption due to prolonged intestinal contact and aggravates the condition. Lactulose should be administered frequently to eliminate the ammonia in the stool. It is an indigestible sugar that acts as an osmotic laxative by increasing the water content of the stool and promoting bowel movements. It is digested by the colonic bacteria, and the acidic remains convert ammonia into ammonium ions in the colon, which are then excreted in the stool. Oral antibiotics can lower blood ammonia levels by decreasing ammonia production and absorption. The commonly used ones are neomycin, metronidazole, vancomycin, and, lately, rifaximin. The last three are better tolerated than neomycin. However antibiotics have their side effects and can cause bacterial overgrowth syndromes. Their main use continues to be in patients who cannot tolerate disaccharides like lactulose. Acarbose and fermentable fiber can also cause decrease in intestinal ammonia production and absorption. Newer studies with sodium benzoate are ongoing. Benzoate and glycine react to form hippurate, and for every mole of benzoate utilized this way, one mole of nitrogen is excreted in the urine, thereby enhancing ammonia metabolism. This, however, still needs to be studied further to be used widely. Studies with ornithine-aspartate are also being done as a stimulator of ammonia metabolism. All the products mentioned are yet to replace lactulose as the first line of treatment but are potentially useful once more studies are done. Other precipitating factors for hepatic encephalopathy include azotemia, hypokalemia, gastrointestinal bleeding, high protein diet, alkalosis, infection, sedatives, and other hepatotoxic agents. High protein diet is a contraindication in this condition, as protein catabolism causes increase in ammonia levels. Daily protein should be restricted to 40 g/day. Sedation of the patient should also be avoided, since sedatives cause cerebral depression and worsening of encephalopathy. These drugs are also not metabolized adequately by the diseased liver. Though infection is an important precipitating factor for hepatic encephalopathy, empiric treatment is not recommended. However, early and adequate treatment of an infection should be done, especially for spontaneous bacterial peritonitis. In fact multiple randomized control trials have been done regarding antibiotic prophylaxis for SBP, and they have shown not only a decrease in bacterial infections but also a significant reduction in mortality. Prophylaxis is recommended in cirrhotic patients with risk factors for SBP like GI bleeding, prior history of SBP, and low ascitic fluid protein. Thiamine and folic acid should be added for nutritional support to all alcoholic patients, since they are malnourished and vitamin depleted. However, this will not change the outcome in hepatic encephalopathy as quickly or as much as avoiding constipation. A 38-year-old woman is going through a divorce and simultaneously filing bankruptcy. She is very stressed about her financial situation and failed marriage. One day, after a particularly long crying spell, she notices a sudden onset of extreme difficulty swallowing at dinnertime. She has difficulty swallowing both solids and liquids. She feels that the food is sticking in her throat. She ignores it. She has numerous bouts of these episodes of difficulty with swallowing. She notices that when she lies down, undigested food comes up. Finally, she sees her doctor. On questioning by her doctor, she insists she has no heartburn. She has not seen any blood when she regurgitates nor has she vomited blood. Manometry is performed which shows an absence of normal peristalsis and an elevated LES pressure. What is the most likely diagnosis? Answer Choices 1 Achalasia 2 Scleroderma 3 Esophageal varices 4 Gastroesophageal reflux disease 5 Mallory-Weiss syndrome - ✔✔Achalasia is the right answer it is characterized by dysphagia. There is poor peristalsis in achalasia. This patient has a characteristic history for achalasia. The onset is during a period of stress. The symptoms consist of dysphagia, complaints of food sticking in the throat, and regurgitation. Scleroderma can present with dysphagia. However, manometry would show a decreased LES pressure, not an elevated LES pressure as the case here. In addition, scleroderma can be associated with heartburn. Esophageal varices usually present with hematemesis. Gastroesophageal reflux disease would present with heartburn. If anything, the LES pressure would be lower than normal, rather than elevated. Mallory-Weiss syndrome is an esophageal tear. This would present with hematemesis. A 60-year-old man presents with severe abdominal pain that started 10 hours ago. It is increasing in severity and is colicky in nature. The patient has not had a bowel movement for 3 days. In the last 2 days, he vomited up what he ate 4 times. He looks tired and dehydrated. On examination, increased bowel sounds were noticed. There is also a mass in the right inguinal area. The patient said he the mass has been present for 10 years but disappears when he lies on his back. Question What is the investigation of choice to confirm the diagnosis? Answer Choices 1 Abdominal X-Ray Erect film 2 Abdominal Ultrasound 3 CBC 4 Colonoscopy 5 Upper GIT endoscopy - ✔✔Abdominal X-ray Erect film. Intestinal obstruction occurs when bowel movement is encountering an obstacle in the passage through the bowel. Actually this can be caused by any mass, adhesion, or any other means of mechanical obstruction. The symptoms of obstruction vary according to the site of obstruction, but generally there is increased bowel movement proximal to obstruction with the reflected increase in bowel sounds. There may be vomiting. There may be no bowel movement, from absolute constipation to feces and flatus. Here the patient has inguinal hernia which is recently irreducible and obstructed. The best diagnostic procedure is erect X-ray film which shows multiple fluid levels and gives confirmation of the diagnosis. The treatment is exploratory surgery, and there is no place for any conservative treatment without surgery. Paralytic ileus is a type of intestinal obstruction that usually follows general anesthesia and is characterized by silent abdomen with no bowel sounds, usually treated by bowel rest and proper hydration. Colonoscopy here is contraindicated. Upper GIT endoscopy is generally used to detect pathology in upper GIT, not in the case of intestinal obstruction. Abdominal ultrasound is of value in diagnosis of gall bladder disease and also urinary bladder pathology. CBC is generally used to detect leukocytosis in acute appendicitis or inflammatory reactions. A 46-year-old woman presents with a 2-month history of heartburn, epigastric discomfort, nausea, and occasional vomiting. She has a history of hyperlipidemia, controlled with diet and exercise, as well as asthma, for which she takes inhalers as needed. She takes no other medications, including over-the-counter analgesics. Family history is noncontributory. On exam she is afebrile, BP120/70 mm Hg, pulse 74/min, and SPO2 92%. Lungs are clear, and she has minimal epigastric tenderness. Otherwise, physical examination is unremarkable. She is advised by her physician to take lansoprazole once daily, which provides only partial relief. Endoscopy is then recommended, which shows a duodenal ulcer. Biopsy reveals infection with Helicobacter pylori. What would be the recommended regimen at this time? Answer Choices 1 Lansoprazole, amoxicillin, and metronidazole twice daily for 2 weeks 2 PPI (proton pump inhibitor), amoxicillin, and clarithromycin twice daily for 2 weeks 3 Omeprazole, tetracycline, and clarithromycin twice daily for 2 weeks 4 Bismuth, metronidazole, and lansoprazole twice daily for 2 weeks 5 Bismuth, metronidazole, and tetracycline 4 times daily for 2 weeks - ✔✔PPI (proton pump inhibitor), amoxicillin, and clarithromycin twice daily for 2 weeks There are several regimens recommended for H. pylori infection, which is an important cause of peptic ulcer disease and should be treated if found associated with symptoms. The choice of the regimen depends on considerations such as cost, side effects, and ease of administration. Allergy to one of the medications, as well as intolerance, should also be taken into account. Any proton pump inhibitor (PPI) with amoxicillin 1000 mg twice daily and clarithromycin 500 mg twice daily for 2 weeks or PPI with metronidazole 500mg twice daily and clarithromycin 500mg twice daily for 2 weeks are recommended. These are the triple drug therapies available. The other regimens suggested are bismuth, metronidazole, and tetracycline 4 times daily for 2 weeks along with PPI twice daily for 2 weeks or H2 receptor antagonist twice daily for 4 weeks (quadruple drug therapy). Dual therapy with a proton pump inhibitor and an antibiotic (amoxicillin or clarithromycin) is not recommended as primary therapy, since eradication rates are much lower than the above regimens. The most common side effect is a metallic taste in the mouth due to clarithromycin or metronidazole. Amoxicillin can cause diarrhea or a rash. Clarithromycin can also cause nausea, vomiting, abdominal pain, and (rarely) QT prolongation. Metronidazole can cause peripheral neuropathy, seizures, and a disulfiram-like reaction when taken with alcohol. Tetracycline is teratogenic and causes photosensitivity. A 64-year-old man presents with a history of progressive dysphagia to solids; recently, he has been having trouble with liquids. He also experiences occasional regurgitation of undigested food. His physical exam is unremarkable. A barium esophagram reveals a distinctive bird's beak appearance of the distal esophagus. Question What is the most likely diagnosis? Answer Choices 1 Gastroparesis 2 Gastroesophageal reflux disease 3 Mallory-Weiss syndrome 4 Achalasia 5 Candidal esophagitis - ✔✔Achalasia The correct answer is achalasia; it often presents with progressive dysphagia to liquids and solids as well as complaints of regurgitation of undigested food. Classic radiographic findings include a bird's beak appearance in the distal esophagus, demonstrating the narrowing of lumen. Gastroparesis is delayed gastric emptying, not an esophageal disorder that would cause the progressive dysphagia noted by the patient. The vagus nerve is not functioning properly in these cases. Gastroesophageal reflux disease can cause dysphagia, but it is not typical; this condition causes regurgitation of undigested food and does not have the classic appearance on esophagram. Mallory-Weiss syndrome presents typically with hematemesis, typically due to forceful vomiting or retching, which causes tears in the esophagus. It is associated with alcoholism. Candidal esophagitis is a fungal infection of the esophagus often seen in immunocompromised patients, such as those who are HIV infected. It presents with progressive dysphagia, but oral thrush is often present in these patients, and an esophagram will not show the classic appearance of achalasia. A 66-year-old man presents with tingling sensations in his hands and feet; his movements are clumsy, and he has difficulty with walking. He notes that he feels generally weak, and that he sometimes has 'the feeling that he can not locate his limbs'. He often feels depressed or irritable. His vitals are normal, but neurological examination reveals decreased reflexes. Question What test should be performed next? Answer Choices 1 Blood tests to measure level of cobalamin 2 Blood tests to measure level of thiamine 3 Blood tests to measure the level of glucose 4 Thyroid-stimulating hormone (TSH) test 5 Neuroimaging studies - ✔✔Blood tests to measure level of cobalamin Blood tests should be performed to measure the level of cobalamin. Based on the symptoms, the patient likely suffers from subacute combined degeneration of spinal cord due to degeneration of the posterior and lateral columns of the spinal cord. This is a result of a vitamin B12 (cobalamin) deficiency; a blood test to determine the level of vitamin B12 should be performed to confirm the diagnosis. Performing blood tests to measure the level of thiamine is an incorrect response. A deficit in vitamin B1 (thiamine) causes symptoms that are characteristic of patients with Wernicke-Korsakoff syndrome. These include confusion and movement disability; however, the patients with Wernicke-Korsakoff syndrome do not generally have tingling or numbness sensations. Performing blood tests to measure the level of glucose is an incorrect response. Symptoms such as pain, tingling, and numbness in the hands and feet, as well as depression, can be signs of developing diabetic neuropathy. This patient lacks other symptoms associated with diabetes, making this diagnosis unlikely; therefore, testing for glucose levels is not a necessary next step. A thyroid-stimulating hormone (TSH) test is an incorrect response. Muscle weakness, numbness, pain, as well as depression, are associated with the late stages of chronic hypothyroidism; however, these symptoms are not sufficient to suspect hypothyroidism. Hypothyroidism is characterized by weight gain, loss of concentration, and hearing loss in the elderly, which are not conditions that affect this patient; therefore, a TSH test would not be useful in this case. Neuroimaging studies are performed to provide information about brain tumors, skull fractures, and other diseases of the brain and blood vessels in the brain. Based on the symptoms seen in this patient, neuroimaging would not help in establishing a diagnosis. A 64-year-old man presents with a history of progressive dysphagia to solids; recently, he has been having trouble with liquids. He also experiences occasional regurgitation of undigested food. His physical exam is unremarkable. A barium esophagram reveals a distinctive bird's beak appearance of the distal esophagus. Question What is the most likely diagnosis? Answer Choices 1 Gastroparesis 2 Gastroesophageal reflux disease 3 Mallory-Weiss syndrome 4 Achalasia 5 Candidal esophagitis - ✔✔Achalasia The correct answer is achalasia; it often presents with progressive dysphagia to liquids and solids as well as complaints of regurgitation of undigested food. Classic radiographic findings include a bird's beak appearance in the distal esophagus, demonstrating the narrowing of lumen. Gastroparesis is delayed gastric emptying, not an esophageal disorder that would cause the progressive dysphagia noted by the patient. The vagus nerve is not functioning properly in these cases. Gastroesophageal reflux disease can cause dysphagia, but it is not typical; this condition causes regurgitation of undigested food and does not have the classic appearance on esophagram. Mallory-Weiss syndrome presents typically with hematemesis, typically due to forceful vomiting or retching, which causes tears in the esophagus. It is associated with alcoholism. Candidal esophagitis is a fungal infection of the esophagus often seen in immunocompromised patients, such as those who are HIV infected. It presents with progressive dysphagia, but oral thrush is often present in these patients, and an esophagram will not show the classic appearance of achalasia. A 58-year-old man presents with a 1-day history of severe abdominal pain, nausea, and vomiting. He initially thought he had some indigestion, with pain located in the epigastric region, and tried some calcium carbonate (Tums) with no relief. The pain and vomiting progressed through the night and kept him from sleeping and going to work. He feels the pain boring through to his back. He denies hematemesis, fever, diarrhea, out-of-the-country travel, and contact with sick people. Prior to onset of pain, he reports good health. He has no known medical conditions and takes no medications. He has had no surgeries. He smokes cigarettes (1ppd x 40 years), admits "moderate" alcohol use, and denies drug use. He is married and works as a welder. Vitals are: BP: 102/56 mmHg; HR: 116bpm; RR: 15; Temp: 98.9F; O2Sat: 95% on room air. On physical exam, the patient appears uncomfortable on the exam table and grimaces when changing position for exam. He is cooperative, alert, and oriented. Abnormal physical exam findings include: abdomen distended, decreased bowel sounds, and tender epigastric region, with guarding. He is tachycardic. No jaundice is noted. The remainder of the exam is normal. This patient's test results are shown in the table. Alk Phosphatase150 (50-136) Amylase 272 (20-110) Lipase 290 (0-160) Question What pharmacologic treatment is the most important intervention for this patient's likely condition? Answer Choices 1 Ertapenem 2 Hyoscyamine 3 Lactated Ringer's 4 Pancrelipase 5 Promethazine - ✔✔LR This patient is presenting with an episode of acute pancreatitis. Pancreatitis is characterized by epigastric pain, nausea, and vomiting. Many other acute abdomen conditions may present similarly. Significant elevations of lipase and amylase, as well as CT evidence of pancreatic inflammation, fluid collections, and/or necrosis, will establish the diagnosis. Pancreatitis can be acute or chronic, and those with chronic disease may have periodic acute flares. In acute pancreatitis, early fluid resuscitation is one of the "few medical interventions that appears to affect outcome," so (of the choices listed) administering lactated Ringer's (or saline) would be the best answer for this patient. Also, ensuring the patient has "nothing by mouth" (NPO) until symptoms decrease is standard treatment. Ertapenem is a carbapenem antibiotic, indicated for complicated intra-abdominal, skin, and urinary tract infections. Although this patient's white blood cell count (WBC) is elevated, the primary process is not infectious. The WBC increases in inflammatory states. Rarely are antibiotics needed in acute pancreatitis. Hyoscyamine is an anticholinergic medication, with a wide range of uses, including many gastrointestinal conditions. It may be used for reducing spasm in irritable bowel syndrome and for biliary colic. Before the test results pointed to pancreatitis, as in this patient's diagnosis, biliary colic and obstruction would be on the differential. However, hyoscyamine would not be a primary treatment for pancreatitis. Pancrelipase is a digestive enzyme, taken with meals, and indicated for pancreatic insufficiency, a common complication of many years of chronic pancreatitis. This patient may be at risk for future pancreatitic disease, but the pancrelipase would not benefit him at this time. Promethazine is a common antiemetic with antihistamine and anticholinergic properties. Although it may be reasonable to consider antiemetic treatment for a patient with acute pancreatitis, it is an adjunct treatment and does not have a vital impact on the course of the disorder. A 58-year-old man presents with a 1-day history of severe abdominal pain, nausea, and vomiting. He initially thought he had some indigestion, with pain located in the epigastric region, and tried some calcium carbonate (Tums) with no relief. The pain and vomiting progressed through the night and kept him from sleeping and going to work. He feels the pain boring through to his back. He denies hematemesis, fever, diarrhea, out-of-the-country travel, and contact with sick people. Prior to onset of pain, he reports good health. He has no known medical conditions and takes no medications. He has had no surgeries. He smokes cigarettes (1ppd x 40 years), admits "moderate" alcohol use, and denies drug use. He is married and works as a welder. Vitals are: BP: 102/56 mmHg; HR: 116bpm; RR: 15; Temp: 98.9F; O2Sat: 95% on room air. On physical exam, the patient appears uncomfortable on the exam table and grimaces when changing position for exam. He is cooperative, alert, and oriented. Abnormal physical exam findings include: abdomen distended, decreased bowel sounds, and tender epigastric region, with guarding. He is tachycardic. No jaundice is noted. The remainder of the exam is normal. This patient's test results are shown in the table. Alk Phosphatase150 (50-136) Amylase 272 (20-110) Lipase 290 (0-160) Question What pharmacologic treatment is the most important intervention for this patient's likely condition? Answer Choices 1 Ertapenem 2 Hyoscyamine 3 Lactated Ringer's 4 Pancrelipase 5 Promethazine - ✔✔LR This patient is presenting with an episode of acute pancreatitis. Pancreatitis is characterized by epigastric pain, nausea, and vomiting. Many other acute abdomen conditions may present similarly. Significant elevations of lipase and amylase, as well as CT evidence of pancreatic inflammation, fluid collections, and/or necrosis, will establish the diagnosis. Pancreatitis can be acute or chronic, and those with chronic disease may have periodic acute flares. In acute pancreatitis, early fluid resuscitation is one of the "few medical interventions that appears to affect outcome," so (of the choices listed) administering lactated Ringer's (or saline) would be the best answer for this patient. Also, ensuring the patient has "nothing by mouth" (NPO) until symptoms decrease is standard treatment. Ertapenem is a carbapenem antibiotic, indicated for complicated intra-abdominal, skin, and urinary tract infections. Although this patient's white blood cell count (WBC) is elevated, the primary process is not infectious. The WBC increases in inflammatory states. Rarely are antibiotics needed in acute pancreatitis. Hyoscyamine is an anticholinergic medication, with a wide range of uses, including many gastrointestinal conditions. It may be used for reducing spasm in irritable bowel syndrome and for biliary colic. Before the test results pointed to pancreatitis, as in this patient's diagnosis, biliary colic and obstruction would be on the differential. However, hyoscyamine would not be a primary treatment for pancreatitis. Pancrelipase is a digestive enzyme, taken with meals, and indicated for pancreatic insufficiency, a common complication of many years of chronic pancreatitis. This patient may be at risk for future pancreatitic disease, but the pancrelipase would not benefit him at this time. Promethazine is a common antiemetic with antihistamine and anticholinergic properties. Although it may be reasonable to consider antiemetic treatment for a patient with acute pancreatitis, it is an adjunct treatment and does not have a vital impact on the course of the disorder. A 58-year-old man presents with a 1-day history of severe abdominal pain, nausea, and vomiting. He initially thought he had some indigestion, with pain located in the epigastric region, and tried some calcium carbonate (Tums) with no relief. The pain and vomiting progressed through the night and kept him from sleeping and going to work. He feels the pain boring through to his back. He denies hematemesis, fever, diarrhea, out-of-the-country travel, and contact with sick people. Prior to onset of pain, he reports good health. He has no known medical conditions and takes no medications. He has had no surgeries. He smokes cigarettes (1ppd x 40 years), admits "moderate" alcohol use, and denies drug use. He is married and works as a welder. Vitals are: BP: 102/56 mmHg; HR: 116bpm; RR: 15; Temp: 98.9F; O2Sat: 95% on room air. On physical exam, the patient appears uncomfortable on the exam table and grimaces when changing position for exam. He is cooperative, alert, and oriented. Abnormal physical exam findings include: abdomen distended, decreased bowel sounds, and tender epigastric region, with guarding. He is tachycardic. No jaundice is noted. The remainder of the exam is normal. This patient's test results are shown in the table. Alk Phosphatase150 (50-136) Amylase 272 (20-110) Lipase 290 (0-160) Question What pharmacologic treatment is the most important intervention for this patient's likely condition? Answer Choices 1 Ertapenem 2 Hyoscyamine 3 Lactated Ringer's 4 Pancrelipase 5 Promethazine - ✔✔LR This patient is presenting with an episode of acute pancreatitis. Pancreatitis is characterized by epigastric pain, nausea, and vomiting. Many other acute abdomen conditions may present similarly. Significant elevations of lipase and amylase, as well as CT evidence of pancreatic inflammation, fluid collections, and/or necrosis, will establish the diagnosis. Pancreatitis can be acute or chronic, and those with chronic disease may have periodic acute flares. In acute pancreatitis, early fluid resuscitation is one of the "few medical interventions that appears to affect outcome," so (of the choices listed) administering lactated Ringer's (or saline) would be the best answer for this patient. Also, ensuring the patient has "nothing by mouth" (NPO) until symptoms decrease is standard treatment. Ertapenem is a carbapenem antibiotic, indicated for complicated intra-abdominal, skin, and urinary tract infections. Although this patient's white blood cell count (WBC) is elevated, the primary process is not infectious. The WBC increases in inflammatory states. Rarely are antibiotics needed in acute pancreatitis. Hyoscyamine is an anticholinergic medication, with a wide range of uses, including many gastrointestinal conditions. It may be used for reducing spasm in irritable bowel syndrome and for biliary colic. Before the test results pointed to pancreatitis, as in this patient's diagnosis, biliary colic and obstruction would be on the differential. However, hyoscyamine would not be a primary treatment for pancreatitis. Pancrelipase is a digestive enzyme, taken with meals, and indicated for pancreatic insufficiency, a common complication of many years of chronic pancreatitis. This patient may be at risk for future pancreatitic disease, but the pancrelipase would not benefit him at this time. Promethazine is a common antiemetic with antihistamine and anticholinergic properties. Although it may be reasonable to consider antiemetic treatment for a patient with acute pancreatitis, it is an adjunct treatment and does not have a vital impact on the course of the disorder. A 58-year-old man presents with a 1-day history of severe abdominal pain, nausea, and vomiting. He initially thought he had some indigestion, with pain located in the epigastric region, and tried some calcium carbonate (Tums) with no relief. The pain and vomiting progressed through the night and kept him from sleeping and going to work. He feels the pain boring through to his back. He denies hematemesis, fever, diarrhea, out-of-the-country travel, and contact with sick people. Prior to onset of pain, he reports good health. He has no known medical conditions and takes no medications. He has had no surgeries. He smokes cigarettes (1ppd x 40 years), admits "moderate" alcohol use, and denies drug use. He is married and works as a welder. Vitals are: BP: 102/56 mmHg; HR: 116bpm; RR: 15; Temp: 98.9F; O2Sat: 95% on room air. On physical exam, the patient appears uncomfortable on the exam table and grimaces when changing position for exam. He is cooperative, alert, and oriented. Abnormal physical exam findings include: abdomen distended, decreased bowel sounds, and tender epigastric region, with guarding. He is tachycardic. No jaundice is noted. The remainder of the exam is normal. This patient's test results are shown in the table. Alk Phosphatase150 (50-136) Amylase 272 (20-110) Lipase 290 (0-160) Question What pharmacologic treatment is the most important intervention for this patient's likely condition? Answer Choices 1 Ertapenem 2 Hyoscyamine 3 Lactated Ringer's 4 Pancrelipase 5 Promethazine - ✔✔LR A 66-year-old man presents with tingling sensations in his hands and feet; his movements are clumsy, and he has difficulty with walking. He notes that he feels generally weak, and that he sometimes has 'the feeling that he can not locate his limbs'. He often feels depressed or irritable. His vitals are normal, but neurological examination reveals decreased reflexes. Question What test should be performed next? Answer Choices 1 Blood tests to measure level of cobalamin 2 Blood tests to measure level of thiamine 3 Blood tests to measure the level of glucose 4 Thyroid-stimulating hormone (TSH) test 5 Neuroimaging studies - ✔✔Blood tests to measure level of cobalamin Blood tests should be performed to measure the level of cobalamin. Based on the symptoms, the patient likely suffers from subacute combined degeneration of spinal cord due to degeneration of the posterior and lateral columns of the spinal cord. This is a result of a vitamin B12 (cobalamin) deficiency; a blood test to determine the level of vitamin B12 should be performed to confirm the diagnosis. Performing blood tests to measure the level of thiamine is an incorrect response. A deficit in vitamin B1 (thiamine) causes symptoms that are characteristic of patients with Wernicke-Korsakoff syndrome. These include confusion and movement disability; however, the patients with Wernicke-Korsakoff syndrome do not generally have tingling or numbness sensations. Performing blood tests to measure the level of glucose is an incorrect response. Symptoms such as pain, tingling, and numbness in the hands and feet, as well as depression, can be signs of developing diabetic neuropathy. This patient lacks other symptoms associated with diabetes, making this diagnosis unlikely; therefore, testing for glucose levels is not a necessary next step. A thyroid-stimulating hormone (TSH) test is an incorrect response. Muscle weakness, numbness, pain, as well as depression, are associated with the late stages of chronic hypothyroidism; however, these symptoms are not sufficient to suspect hypothyroidism. Hypothyroidism is characterized by weight gain, loss of concentration, and hearing loss in the elderly, which are not conditions that affect this patient; therefore, a TSH test would not be useful in this case. Neuroimaging studies are performed to provide information about brain tumors, skull fractures, and other diseases of the brain and blood vessels in the brain. Based on the symptoms seen in this patient, neuroimaging would not help in establishing a diagnosis. A 64-year-old man presents with a history of progressive dysphagia to solids; recently, he has been having trouble with liquids. He also experiences occasional regurgitation of undigested food. His physical exam is unremarkable. A barium esophagram reveals a distinctive bird's beak appearance of the distal esophagus. Question What is the most likely diagnosis? Answer Choices 1 Gastroparesis 2 Gastroesophageal reflux disease 3 Mallory-Weiss syndrome 4 Achalasia 5 Candidal esophagitis - ✔✔Achalasia The correct answer is achalasia; it often presents with progressive dysphagia to liquids and solids as well as complaints of regurgitation of undigested food. Classic radiographic findings include a bird's beak appearance in the distal esophagus, demonstrating the narrowing of lumen. Gastroparesis is delayed gastric emptying, not an esophageal disorder that would cause the progressive dysphagia noted by the patient. The vagus nerve is not functioning properly in these cases. Gastroesophageal reflux disease can cause dysphagia, but it is not typical; this condition causes regurgitation of undigested food and does not have the classic appearance on esophagram. Mallory-Weiss syndrome presents typically with hematemesis, typically due to forceful vomiting or retching, which causes tears in the esophagus. It is associated with alcoholism. Candidal esophagitis is a fungal infection of the esophagus often seen in immunocompromised patients, such as those who are HIV infected. It presents with progressive dysphagia, but oral thrush is often present in these patients, and an esophagram will not show the classic appearance of achalasia. A 66-year-old man presents with tingling sensations in his hands and feet; his movements are clumsy, and he has difficulty with walking. He notes that he feels generally weak, and that he sometimes has 'the feeling that he can not locate his limbs'. He often feels depressed or irritable. His vitals are normal, but neurological examination reveals decreased reflexes. Question What test should be performed next? Answer Choices 1 Blood tests to measure level of cobalamin 2 Blood tests to measure level of thiamine 3 Blood tests to measure the level of glucose 4 Thyroid-stimulating hormone (TSH) test 5 Neuroimaging studies - ✔✔Blood tests to measure level of cobalamin Blood tests should be performed to measure the level of cobalamin. Based on the symptoms, the patient likely suffers from subacute combined degeneration of spinal cord due to degeneration of the posterior and lateral columns of the spinal cord. This is a result of a vitamin B12 (cobalamin) deficiency; a blood test to determine the level of vitamin B12 should be performed to confirm the diagnosis. Performing blood tests to measure the level of thiamine is an incorrect response. A deficit in vitamin B1 (thiamine) causes symptoms that are characteristic of patients with Wernicke-Korsakoff syndrome. These include confusion and movement disability; however, the patients with Wernicke-Korsakoff syndrome do not generally have tingling or numbness sensations. Performing blood tests to measure the level of glucose is an incorrect response. Symptoms such as pain, tingling, and numbness in the hands and feet, as well as depression, can be signs of developing diabetic neuropathy. This patient lacks other symptoms associated with diabetes, making this diagnosis unlikely; therefore, testing for glucose levels is not a necessary next step. A thyroid-stimulating hormone (TSH) test is an incorrect response. Muscle weakness, numbness, pain, as well as depression, are associated with the late stages of chronic hypothyroidism; however, these symptoms are not sufficient to suspect hypothyroidism. Hypothyroidism is characterized by weight gain, loss of concentration, and hearing loss in the elderly, which are not conditions that affect this patient; therefore, a TSH test would not be useful in this case. Neuroimaging studies are performed to provide information about brain tumors, skull fractures, and other diseases of the brain and blood vessels in the brain. Based on the symptoms seen in this patient, neuroimaging would not help in establishing a diagnosis. A 64-year-old man presents with a history of progressive dysphagia to solids; recently, he has been having trouble with liquids. He also experiences occasional regurgitation of undigested food. His physical exam is unremarkable. A barium esophagram reveals a distinctive bird's beak appearance of the distal esophagus. Question What is the most likely diagnosis? Answer Choices 1 Gastroparesis 2 Gastroesophageal reflux disease 3 Mallory-Weiss syndrome 4 Achalasia 5 Candidal esophagitis - ✔✔Achalasia The correct answer is achalasia; it often presents with progressive dysphagia to liquids and solids as well as complaints of regurgitation of undigested food. Classic radiographic findings include a bird's beak appearance in the distal esophagus, demonstrating the narrowing of lumen. Gastroparesis is delayed gastric emptying, not an esophageal disorder that would cause the progressive dysphagia noted by the patient. The vagus nerve is not functioning properly in these cases. Gastroesophageal reflux disease can cause dysphagia, but it is not typical; this condition causes regurgitation of undigested food and does not have the classic appearance on esophagram. Mallory-Weiss syndrome presents typically with hematemesis, typically due to forceful vomiting or retching, which causes tears in the esophagus. It is associated with alcoholism. Candidal esophagitis is a fungal infection of the esophagus often seen in immunocompromised patients, such as those who are HIV infected. It presents with progressive dysphagia, but oral thrush is often present in these patients, and an esophagram will not show the classic appearance of achalasia. A 64-year-old man presents with a history of progressive dysphagia to solids; recently, he has been having trouble with liquids. He also experiences occasional regurgitation of undigested food. His physical exam is unremarkable. A barium esophagram reveals a distinctive bird's beak appearance of the distal esophagus. Question What is the most likely diagnosis? Answer Choices 1 Gastroparesis 2 Gastroesophageal reflux disease 3 Mallory-Weiss syndrome 4 Achalasia 5 Candidal esophagitis - ✔✔Achalasia The correct answer is achalasia; it often presents with progressive dysphagia to liquids and solids as well as complaints of regurgitation of undigested food. Classic radiographic findings include a bird's beak appearance in the distal esophagus, demonstrating the narrowing of lumen. Gastroparesis is delayed gastric emptying, not an esophageal disorder that would cause the progressive dysphagia noted by the patient. The vagus nerve is not functioning properly in these cases. Gastroesophageal reflux disease can cause dysphagia, but it is not typical; this condition causes regurgitation of undigested food and does not have the classic appearance on esophagram. Mallory-Weiss syndrome presents typically with hematemesis, typically due to forceful vomiting or retching, which causes tears in the esophagus. It is associated with alcoholism. Candidal esophagitis is a fungal infection of the esophagus often seen in immunocompromised patients, such as those who are HIV infected. It presents with progressive dysphagia, but oral thrush is often present in these patients, and an esophagram will not show the classic appearance of achalasia. A 64-year-old man presents with a history of progressive dysphagia to solids; recently, he has been having trouble with liquids. He also experiences occasional regurgitation of undigested food. His physical exam is unremarkable. A barium esophagram reveals a distinctive bird's beak appearance of the distal esophagus. Question What is the most likely diagnosis? Answer Choices 1 Gastroparesis 2 Gastroesophageal reflux disease 3 Mallory-Weiss syndrome 4 Achalasia 5 Candidal esophagitis - ✔✔ A 46-year-old woman presents with a 2-month history of heartburn, epigastric discomfort, nausea, and occasional vomiting. She has a history of hyperlipidemia, controlled with diet and exercise, as well as asthma, for which she takes inhalers as needed. She takes no other medications, including over-the-counter analgesics. Family history is noncontributory. On exam she is afebrile, BP120/70 mm Hg, pulse 74/min, and SPO2 92%. Lungs are clear, and she has minimal epigastric tenderness. Otherwise, physical examination is unremarkable. She is advised by her physician to take lansoprazole once daily, which provides only partial relief. Endoscopy is then recommended, which shows a duodenal ulcer. Biopsy reveals infection with Helicobacter pylori. What would be the recommended regimen at this time? Answer Choices 1 Lansoprazole, amoxicillin, and metronidazole twice daily for 2 weeks 2 PPI (proton pump inhibitor), amoxicillin, and clarithromycin twice daily for 2 weeks 3 Omeprazole, tetracycline, and clarithromycin twice daily for 2 weeks 4 Bismuth, metronidazole, and lansoprazole twice daily for 2 weeks 5 Bismuth, metronidazole, and tetracycline 4 times daily for 2 weeks - ✔✔PPI (proton pump inhibitor), amoxicillin, and clarithromycin twice daily for 2 weeks There are several regimens recommended for H. pylori infection, which is an important cause of peptic ulcer disease and should be treated if found associated with symptoms. The choice of the regimen depends on considerations such as cost, side effects, and ease of administration. Allergy to one of the medications, as well as intolerance, should also be taken into account. Any proton pump inhibitor (PPI) with amoxicillin 1000 mg twice daily and clarithromycin 500 mg twice daily for 2 weeks or PPI with metronidazole 500mg twice daily and clarithromycin 500mg twice daily for 2 weeks are recommended. These are the triple drug therapies available. The other regimens suggested are bismuth, metronidazole, and tetracycline 4 times daily for 2 weeks along with PPI twice daily for 2 weeks or H2 receptor antagonist twice daily for 4 weeks (quadruple drug therapy). Dual therapy with a proton pump inhibitor and an antibiotic (amoxicillin or clarithromycin) is not recommended as primary therapy, since eradication rates are much lower than the above regimens. The most common side effect is a metallic taste in the mouth due to clarithromycin or metronidazole. Amoxicillin can cause diarrhea or a rash. Clarithromycin can also cause nausea, vomiting, abdominal pain, and (rarely) QT prolongation. Metronidazole can cause peripheral neuropathy, seizures, and a disulfiram-like reaction when taken with alcohol. Tetracycline is teratogenic and causes photosensitivity. A 46-year-old woman presents with a 2-month history of heartburn, epigastric discomfort, nausea, and occasional vomiting. She has a history of hyperlipidemia, controlled with diet and exercise, as well as asthma, for which she takes inhalers as needed. She takes no other medications, including over-the-counter analgesics. Family history is noncontributory. On exam she is afebrile, BP120/70 mm Hg, pulse 74/min, and SPO2 92%. Lungs are clear, and she has minimal epigastric tenderness. Otherwise, physical examination is unremarkable. She is advised by her physician to take lansoprazole once daily, which provides only partial relief. Endoscopy is then recommended, which shows a duodenal ulcer. Biopsy reveals infection with Helicobacter pylori. What would be the recommended regimen at this time? Answer Choices 1 Lansoprazole, amoxicillin, and metronidazole twice daily for 2 weeks 2 PPI (proton pump inhibitor), amoxicillin, and clarithromycin twice daily for 2 weeks 3 Omeprazole, tetracycline, and clarithromycin twice daily for 2 weeks 4 Bismuth, metronidazole, and lansoprazole twice daily for 2 weeks 5 Bismuth, metronidazole, and tetracycline 4 times daily for 2 weeks - ✔✔PPI (proton pump inhibitor), amoxicillin, and clarithromycin twice daily for 2 weeks A 21-year-old man is brought to the emergency room by his friends. One hour earlier he started having excessive retching and he vomited violently several times. He has been drinking for the last 3 hours. The vomitus was mainly bright red blood (about 2 quarts) and dizziness followed. What is the likely causer of this clinical picture? Answer Choices 1 Acute gastritis 2 Carcinoma of stomach 3 Acute esophagitis 4 A gastroesophageal mucosal tear 5 Rupture of the lower esophagus - ✔✔A gastroesophageal mucosal tear This is a classic presentation of Mallory-Weiss syndrome. Vomiting and retching may cause a tear that involves only the mucosa and is not transmural. The tear usually involves the gastric mucosa near the squamous to columnar mucosal junction, but it may also involve the esophageal mucosa. A wide variety of causes such as gastroenteritis, alcohol binging, ulcers, and hiatus hernia have been cited. In fact, any condition which causes retching and vomiting can cause the tear. Patients present with upper gastrointestinal bleeding that may be severe. Most patients recover with only conservative management, but those with severe arterial bleeding require surgery. A 21-year-old man is brought to the emergency room by his friends. One hour earlier he started having excessive retching and he vomited violently several times. He has been drinking for the last 3 hours. The vomitus was mainly bright red blood (about 2 quarts) and dizziness followed. What is the likely causer of this clinical picture? Answer Choices 1 Acute gastritis 2 Carcinoma of stomach 3 Acute esophagitis 4 A gastroesophageal mucosal tear 5 Rupture of the lower esophagus - ✔✔A gastroesophageal mucosal tear This is a classic presentation of Mallory-Weiss syndrome. Vomiting and retching may cause a tear that involves only the mucosa and is not transmural. The tear usually involves the gastric mucosa near the squamous to columnar mucosal junction, but it may also involve the esophageal mucosa. A wide variety of causes such as gastroenteritis, alcohol binging, ulcers, and hiatus hernia have been cited. In fact, any condition which causes retching and vomiting can cause the tear. Patients present with upper gastrointestinal bleeding that may be severe. Most patients recover with only conservative management, but those with severe arterial bleeding require surgery. A 21-year-old man is brought to the emergency room by his friends. One hour earlier he started having excessive retching and he vomited violently several times. He has been drinking for the last 3 hours. The vomitus was mainly bright red blood (about 2 quarts) and dizziness followed. What is the likely causer of this clinical picture? Answer Choices 1 Acute gastritis 2 Carcinoma of stomach 3 Acute esophagitis 4 A gastroesophageal mucosal tear 5 Rupture of the lower esophagus - ✔✔ A 15-year-old female adolescent is brought into the emergency department by her mother with a history of ingesting an unknown number of acetaminophen tablets within the past hour. Neither the mother nor the daughter had any idea how many pills had previously been used or were ingested. The daughter stated she took a lot of pills. The mother stated her daughter had no other medical problems. Both the mother and daughter said that no other medications were being taken. What is the most appropriate time to evaluate an acetaminophen level after the reported ingestion in this patient to determine if therapy is indicated? Answer Choices 1 8 hours 2 4 hours 3 12 hours 4 24 hours 5 2 hours - ✔✔4 hrs In an acute ingestion, the peak concentration may not be achieved until 4 hoursafter ingestion. Absorption may be affected by coingestants that affect gastric motility. Concretions of multiple tablets ingested at the same time may form bezoars in the stomach and alter absorption, or provide a continuing source of supply. A single acetaminophen level drawn at least 4 hours after ingestion is sufficient to determine patient management. If time of ingestion is unknown, an immediate level and at least 2 additional levels drawn 4 hours apart may be useful. Clinical toxicity may not be evident soon after overdose, and the risk of morbidity increases when the initiation of therapy is delayed. At therapeutic doses, peak concentration is generally achieved after 1 hour, with half-life of 2 to 4 hours. The toxic dose of acetaminophen is approximately 150 mg/kg or 7 grams in adults. Children may tolerate doses up to 200 mg/kg. Acetaminophen is rapidly absorbed from the stomach and small intestine and metabolized by conjugation in the liver to nontoxic agents. These water-soluble conjugates are then eliminated in the urine. Ninety percent are metabolized to either metabolites of sulfate (primarily in children) or glucuronide (primarily in adults). Approximately 4% is metabolized by the cytochrome p450 system to an active metabolite, NAPQI (n-acetyl-p-benzoquinoneimine). Normally, NAPQI is conjugated with glutathione resulting in detoxification and excreted as mercapturic acid and cysteine conjugates. In acute ingestion, glutathione is depleted, so NAPQI covalently binds to vital proteins and the lipid bilayer of hepatocyte membranes. The result is hepatocellular death and centrilobular liver necrosis. Other organs may be affected. The acetaminophen level should be plotted on the Rumack-Matthew nomogram, a semi-logarithmic plot of the acetaminophen level over time (4-24 hours post ingestion) based on adult data. Approximately 60% of patients with levels above the line will develop serum transaminases >1000 IU/L. A second line 25% below the first line was added. Levels between the first and second line are considered possibly toxic and take into account such factors as lab error and error in time of ingestion. There is no clinical evidence suggesting that patients in this range need to be treated. Levels below the second line are low risk. The nomogram is not valid for ingestions less than 4 hours, chronic ingestions, or for patients who have ingested toxic doses of extended release preparations. Because the absorption of extended release preparations is delayed, a measurement at 4 hours will not accurately reflect absorption. The nomogram is also not valid for patients with chronic alcohol consumption. A 63-year-old woman presents with a 6-month history of worsening difficulty in swallowing. Shortly after swallowing, she feels like something is getting stuck in her upper chest, and the sensation lasts long enough to begin causing significant chest discomfort just behind her breastbone. The difficulty swallowing is often extremely variable and intermittent; it has not been progressive. It gets to the point that she feels like she is going to regurgitate her food, and she is also experiencing substantial episodes of acid reflux. The difficulty swallowing seems to be worsened by when the patient is extremely stressed and when she eats hot or cold food. She denies any weight loss, night sweats, or other significant issues. Physical examination of the patient is otherwise noncontributory. An extensive gastrointestinal evaluation, which included a comprehensive endoscopic evaluation, is negative. Question What is the first-line prescribed medication that would help alleviate this patient's signs and symptoms? Answer Choices 1 Injected Botulinum toxin 2 Intravenous nitroglycerin 3 Oral proton pump inhibitor (PPI) 4 Oral prednisone 5 Oral Metoclopramide - ✔✔Oral PPI The correct response is oral proton pump inhibitor. The patient in this presented scenario is showing signs and symptoms that are most consistent with esophageal dysphagia. Difficulty swallowing is the main complaint and is typically caused by 1 of 2 entities: localized neuromuscular disorders or obstructive lesions. Our patient specifically is showing issues of diffuse esophageal spasm. This is a motility disorder that is defined by simultaneous uncoordinated contraction of several esophageal segments. This can lead to potentially retrosternal pain and is worsened or precipitated by acid reflex, rapid eating, stress/anxiety, and hot/cold food. The dysphagia found in these patients is intermittent, non-progressive and does not cause weight loss. These patients have no documented abnormality in the distribution of myenteric neurons, normal lower esophageal sphincter relaxation, and no evidence of obstruction. All of these components are consistent with the patient described. In terms of a first-line treatment, acid suppression with a proton pump inhibitor (PPI) is the first-line pharmaceutical intervention that should be initiated. Sublingual nitroglycerin and calcium channel blockers may also be considered as treatment, but they are not typically considered first-line options. Botulinum toxin is not considered first-line treatment for diffuse esophageal spasms; it may be considered to be administered/injected in the lower esophageal sphincter and lower esophagus to reduce chest pain caused by diffuse esophageal spasm but again is not the first line treatment. Also, the patient described above is experiencing other symptoms, not just the chest pain. Prokinetic agents, such as metoclopramide, are considered treatment options for reflux esophagitis and Barrett esophagus-related causes of dysphagia. Oral corticosteroids may be considered in patients with diagnosed eosinophilic esophagitis. A 62-year-old man passed a foul-smelling, tarry stool 1 week ago. 3 weeks ago, he began taking diclofenac because of worsening osteoarthritis. His past medical history includes primary hypertension, coronary heart disease, and 2 episodes of venous thrombosis in the calf. Besides diclofenac, his medications include warfarin, enalapril, low-dose aspirin, propranolol, and sublingual nitroglycerin. A screening colonoscopy performed 2 weeks ago was entirely normal. Vital signs are stable, and the physical examination is unremarkable. Upper endoscopy shows only pangastritis and erosions. What is the most effective way of reducing his risk of rebleeding? Answer Choices 1 Misoprostol 2 Omeprazole 3 Famotidine 4 Switch to valdecoxib 5 Take the diclofenac tablets with at least 250 ml of water - ✔✔Misoprostol The correct response is misoprostol. Besides NSAID use, this patient has several risk factors for gastric bleeding (a previous episode, advanced age, use of anticoagulants and low-dose aspirin). Therefore, prophylaxis against bleeding must be seriously considered. Prostaglandin analogues are the most effective drugs for reducing the risk of rebleeding. One study described reductions of up to fivefold in the incidence of gastric ulcers on endoscopy. However, this drug requires 2 to 4 daily doses and has several side effects. The most common are diarrhea (up to 30% of patients) and abdominal discomfort. It is also a potent abortifacient. Starting the drug at a low dose and then progressively raising it might reduce the incidence of these side effects and improve tolerance. Proton-pump inhibitors such as omeprazole also reduce the incidence of gastrointestinal complications in chronic users of NSAIDs. They are slightly less effective than prostaglandin analogues, but their once-daily dosage is more convenient and the incidence of side effects is lower. The use of low-dose aspirin abrogates the protective effects of COX-2-selective anti-inflammatory drugs (i.e., celecoxib, rofecoxib, or valdecoxib). Therefore, switching to COX-2 inhibitors would not be advantageous for this patient. H2-blockers such as famotidine have only marginal efficacy in reducing the incidence of NSAID-associated gastric ulcers and should not be prescribed in this case. The medications that should be taken with at least 250 ml of water are the bisphosphonates. This precaution reduces the risk of pill esophagitis. This drug should also be taken on an empty stomach and with the patient in an upright position. If these precautions are not followed, bisphosphonates can cause severe esophagitis, which can even lead to strictures or bleeding. A 63-year-old woman presents with a 6-month history of worsening difficulty in swallowing. Shortly after swallowing, she feels like something is getting stuck in her upper chest, and the sensation lasts long enough to begin causing significant chest discomfort just behind her breastbone. The difficulty swallowing is often extremely variable and intermittent; it has not been progressive. It gets to the point that she feels like she is going to regurgitate her food, and she is also experiencing substantial episodes of acid reflux. The difficulty swallowing seems to be worsened by when the patient is extremely stressed and when she eats hot or cold food. She denies any weight loss, night sweats, or other significant issues. Physical examination of the patient is otherwise noncontributory. An extensive gastrointestinal evaluation, which included a comprehensive endoscopic evaluation, is negative. Question What is the first-line prescribed medication that would help alleviate this patient's signs and symptoms? Answer Choices 1 Injected Botulinum toxin 2 Intravenous nitroglycerin 3 Oral proton pump inhibitor (PPI) 4 Oral prednisone 5 Oral Metoclopramide - ✔✔Oral PPI The correct response is oral proton pump inhibitor. The patient in this presented scenario is showing signs and symptoms that are most consistent with esophageal dysphagia. Difficulty swallowing is the main complaint and is typically caused by 1 of 2 entities: localized neuromuscular disorders or obstructive lesions. Our patient specifically is showing issues of diffuse esophageal spasm. This is a motility disorder that is defined by simultaneous uncoordinated contraction of several esophageal segments. This can lead to potentially retrosternal pain and is worsened or precipitated by acid reflex, rapid eating, stress/anxiety, and hot/cold food. The dysphagia found in these patients is intermittent, non-progressive and does not cause weight loss. These patients have no documented abnormality in the distribution of myenteric neurons, normal lower esophageal sphincter relaxation, and no evidence of obstruction. All of these components are consistent with the patient described. In terms of a first-line treatment, acid suppression with a proton pump inhibitor (PPI) is the first-line pharmaceutical intervention that should be initiated. Sublingual nitroglycerin and calcium channel blockers may also be considered as treatment, but they are not typically considered first-line options. Botulinum toxin is not considered first-line treatment for diffuse esophageal spasms; it may be considered to be administered/injected in the lower esophageal sphincter and lower esophagus to reduce chest pain caused by diffuse esophageal spasm but again is not the first line treatment. Also, the patient described above is experiencing other symptoms, not just the chest pain. Prokinetic agents, such as metoclopramide, are considered treatment options for reflux esophagitis and Barrett esophagus-related causes of dysphagia. Oral corticosteroids may be considered in patients with diagnosed eosinophilic esophagitis. A 63-year-old woman presents with a 6-month history of worsening difficulty in swallowing. Shortly after swallowing, she feels like something is getting stuck in her upper chest, and the sensation lasts long enough to begin causing significant chest discomfort just behind her breastbone. The difficulty swallowing is often extremely variable and intermittent; it has not been progressive. It gets to the point that she feels like she is going to regurgitate her food, and she is also experiencing substantial episodes of acid reflux. The difficulty swallowing seems to be worsened by when the patient is extremely stressed and when she eats hot or cold food. She denies any weight loss, night sweats, or other significant issues. Physical examination of the patient is otherwise noncontributory. An extensive gastrointestinal evaluation, which included a comprehensive endoscopic evaluation, is negative. Question What is the first-line prescribed medication that would help alleviate this patient's signs and symptoms? Answer Choices 1 Injected Botulinum toxin 2 Intravenous nitroglycerin 3 Oral proton pump inhibitor (PPI) 4 Oral prednisone 5 Oral Metoclopramide - ✔✔Oral PPI The correct response is oral proton pump inhibitor. The patient in this presented scenario is showing signs and symptoms that are most consistent with esophageal dysphagia. Difficulty swallowing is the main complaint and is typically caused by 1 of 2 entities: localized neuromuscular disorders or obstructive lesions. Our patient specifically is showing issues of diffuse esophageal spasm. This is a motility disorder that is defined by simultaneous uncoordinated contraction of several esophageal segments. This can lead to potentially retrosternal pain and is worsened or precipitated by acid reflex, rapid eating, stress/anxiety, and hot/cold food. The dysphagia found in these patients is intermittent, non-progressive and does not cause weight loss. These patients have no documented abnormality in the distribution of myenteric neurons, normal lower esophageal sphincter relaxation, and no evidence of obstruction. All of these components are consistent with the patient described. In terms of a first-line treatment, acid suppression with a proton pump inhibitor (PPI) is the first-line pharmaceutical intervention that should be initiated. Sublingual nitroglycerin and calcium channel blockers may also be considered as treatment, but they are not typically considered first-line options. Botulinum toxin is not considered first-line treatment for diffuse esophageal spasms; it may be considered to be administered/injected in the lower esophageal sphincter and lower esophagus to reduce chest pain caused by diffuse esophageal spasm but again is not the first line treatment. Also, the patient described above is experiencing other symptoms, not just the chest pain. Prokinetic agents, such as metoclopramide, are considered treatment options for reflux esophagitis and Barrett esophagus-related causes of dysphagia. Oral corticosteroids may be considered in patients with diagnosed eosinophilic esophagitis. A 63-year-old woman presents with a 6-month history of worsening difficulty in swallowing. Shortly after swallowing, she feels like something is getting stuck in her upper chest, and the sensation lasts long enough to begin causing significant chest discomfort just behind her breastbone. The difficulty swallowing is often extremely variable and intermittent; it has not been progressive. It gets to the point that she feels like she is going to regurgitate her food, and she is also experiencing substantial episodes of acid reflux. The difficulty swallowing seems to be worsened by when the patient is extremely stressed and when she eats hot or cold food. She denies any weight loss, night sweats, or other significant issues. Physical examination of the patient is otherwise noncontributory. An extensive gastrointestinal evaluation, which included a comprehensive endoscopic evaluation, is negative. Question What is the first-line prescribed medication that would help alleviate this patient's signs and symptoms? Answer Choices 1 Injected Botulinum toxin 2 Intravenous nitroglycerin 3 Oral proton pump inhibitor (PPI) 4 Oral prednisone 5 Oral Metoclopramide - ✔✔Oral PPI A 15-year-old boy was healthy until 3 months ago. He has been having episodes of crampy abdominal pain and explosive liquid stools 2-3 times a day. He has had intermittent fevers and has noted blood in the stool. He has a documented 15-lb weight loss. Question What is the best way to start treatment? Answer Choices 1 Diphenoxylate hydrochloride with atropine sulfate 2 Eliminate lactose-containing foods 3 Loperamide HCL 4 Eliminate all high fiber foods 5 Sulfasalazine - ✔✔Sulfasalazine This patient has ulcerative colitis. Therefore, it would be most appropriate to start treatment with sulfasalazine at 50-75 mg/kg/day in 2-4 divided doses. Sulfasalazine is a sulfa drug that has important anti-inflammatory properties because it inhibits the synthesis of mediators of the inflammatory response. In some patients, sulfasalazine is poorly tolerated, in which case mesalamine (50-100 mg/kg/day) and balsalazide (110-175 mg/kg/day) are preferable treatments. Diphenoxylate hydrochloride with atropine sulfate at 1-2 mL t.i.d. and loperamide HCl at 0.04-0.08 mg/kg/day in 2-4 divided doses are contraindicated because they would suppress the symptoms of ulcerative colitis without amelioration of the underlying cause of the condition. Elimination of lactose-containing food would have no effect because the patient is lactose-tolerant. In addition, elimination of dietary fiber would not have any effect. A 15-year-old female adolescent is brought into the emergency department by her mother with a history of ingesting an unknown number of acetaminophen tablets within the past hour. Neither the mother nor the daughter had any idea how many pills had previously been used or were ingested. The daughter stated she took a lot of pills. The mother stated her daughter had no other medical problems. Both the mother and daughter said that no other medications were being taken. What is the most appropriate time to evaluate an acetaminophen level after the reported ingestion in this patient to determine if therapy is indicated? Answer Choices 1 8 hours 2 4 hours 3 12 hours 4 24 hours 5 2 hours - ✔✔4 hrs In an acute ingestion, the peak concentration may not be achieved until 4 hoursafter ingestion. Absorption may be affected by coingestants that affect gastric motility. Concretions of multiple tablets ingested at the same time may form bezoars in the stomach and alter absorption, or provide a continuing source of supply. A single acetaminophen level drawn at least 4 hours after ingestion is sufficient to determine patient management. If time of ingestion is unknown, an immediate level and at least 2 additional levels drawn 4 hours apart may be useful. Clinical toxicity may not be evident soon after overdose, and the risk of morbidity increases when the initiation of therapy is delayed. At therapeutic doses, peak concentration is generally achieved after 1 hour, with half-life of 2 to 4 hours. The toxic dose of acetaminophen is approximately 150 mg/kg or 7 grams in adults. Children may tolerate doses up to 200 mg/kg. Acetaminophen is rapidly absorbed from the stomach and small intestine and metabolized by conjugation in the liver to nontoxic agents. These water-soluble conjugates are then eliminated in the urine. Ninety percent are metabolized to either metabolites of sulfate (primarily in children) or glucuronide (primarily in adults). Approximately 4% is metabolized by the cytochrome p450 system to an active metabolite, NAPQI (n-acetyl-p-benzoquinoneimine). Normally, NAPQI is conjugated with glutathione resulting in detoxification and excreted as mercapturic acid and cysteine conjugates. In acute ingestion, glutathione is depleted, so NAPQI covalently binds to vital proteins and the lipid bilayer of hepatocyte membranes. The result is hepatocellular death and centrilobular liver necrosis. Other organs may be affected. The acetaminophen level should be plotted on the Rumack-Matthew nomogram, a semi-logarithmic plot of the acetaminophen level over time (4-24 hours post ingestion) based on adult data. Approximately 60% of patients with levels above the line will develop serum transaminases >1000 IU/L. A second line 25% below the first line was added. Levels between the first and second line are considered possibly toxic and take into account such factors as lab error and error in time of ingestion. There is no clinical evidence suggesting that patients in this range need to be treated. Levels below the second line are low risk. The nomogram is not valid for ingestions less than 4 hours, chronic ingestions, or for patients who have ingested toxic doses of extended release preparations. Because the absorption of extended release preparations is delayed, a measurement at 4 hours will not accurately reflect absorption. The nomogram is also not valid for patients with chronic alcohol consumption. A 15-year-old boy was healthy until 3 months ago. He has been having episodes of crampy abdominal pain and explosive liquid stools 2-3 times a day. He has had intermittent fevers and has noted blood in the stool. He has a documented 15-lb weight loss. Question What is the best way to start treatment? Answer Choices 1 Diphenoxylate hydrochloride with atropine sulfate 2 Eliminate lactose-containing foods 3 Loperamide HCL 4 Eliminate all high fiber foods 5 Sulfasalazine - ✔✔Sulfasalazine This patient has ulcerative colitis. Therefore, it would be most appropriate to start treatment with sulfasalazine at 50-75 mg/kg/day in 2-4 divided doses. Sulfasalazine is a sulfa drug that has important anti-inflammatory properties because it inhibits the synthesis of mediators of the inflammatory response. In some patients, sulfasalazine is poorly tolerated, in which case mesalamine (50-100 mg/kg/day) and balsalazide (110-175 mg/kg/day) are preferable treatments. Diphenoxylate hydrochloride with atropine sulfate at 1-2 mL t.i.d. and loperamide HCl at 0.04-0.08 mg/kg/day in 2-4 divided doses are contraindicated because they would suppress the symptoms of ulcerative colitis without amelioration of the underlying cause of the condition. Elimination of lactose-containing food would have no effect because the patient is lactose-tolerant. In addition, elimination of dietary fiber would not have any effect. A 15-year-old female adolescent is brought into the emergency department by her mother with a history of ingesting an unknown number of acetaminophen tablets within the past hour. Neither the mother nor the daughter had any idea how many pills had previously been used or were ingested. The daughter stated she took a lot of pills. The mother stated her daughter had no other medical problems. Both the mother and daughter said that no other medications were being taken. What is the most appropriate time to evaluate an acetaminophen level after the reported ingestion in this patient to determine if therapy is indicated? Answer Choices 1 8 hours 2 4 hours 3 12 hours 4 24 hours 5 2 hours - ✔✔4 hrs In an acute ingestion, the peak concentration may not be achieved until 4 hoursafter ingestion. Absorption may be affected by coingestants that affect gastric motility. Concretions of multiple tablets ingested at the same time may form bezoars in the stomach and alter absorption, or provide a continuing source of supply. A single acetaminophen level drawn at least 4 hours after ingestion is sufficient to determine patient management. If time of ingestion is unknown, an immediate level and at least 2 additional levels drawn 4 hours apart may be useful. Clinical toxicity may not be evident soon after overdose, and the risk of morbidity increases when the initiation of therapy is delayed. At therapeutic doses, peak concentration is generally achieved after 1 hour, with half-life of 2 to 4 hours. The toxic dose of acetaminophen is approximately 150 mg/kg or 7 grams in adults. Children may tolerate doses up to 200 mg/kg. Acetaminophen is rapidly absorbed from the stomach and small intestine and metabolized by conjugation in the liver to nontoxic agents. These water-soluble conjugates are then eliminated in the urine. Ninety percent are metabolized to either metabolites of sulfate (primarily in children) or glucuronide (primarily in adults). Approximately 4% is metabolized by the cytochrome p450 system to an active metabolite, NAPQI (n-acetyl-p-benzoquinoneimine). Normally, NAPQI is conjugated with glutathione resulting in detoxification and excreted as mercapturic acid and cysteine conjugates. In acute ingestion, glutathione is depleted, so NAPQI covalently binds to vital proteins and the lipid bilayer of hepatocyte membranes. The result is hepatocellular death and centrilobular liver necrosis. Other organs may be affected. The acetaminophen level should be plotted on the Rumack-Matthew nomogram, a semi-logarithmic plot of the acetaminophen level over time (4-24 hours post ingestion) based on adult data. Approximately 60% of patients with levels above the line will develop serum transaminases >1000 IU/L. A second line 25% below the first line was added. Levels between the first and second line are considered possibly toxic and take into account such factors as lab error and error in time of ingestion. There is no clinical evidence suggesting that patients in this range need to be treated. Levels below the second line are low risk. The nomogram is not valid for ingestions less than 4 hours, chronic ingestions, or for patients who have ingested toxic doses of extended release preparations. Because the absorption of extended release preparations is delayed, a measurement at 4 hours will not accurately reflect absorption. The nomogram is also not valid for patients with chronic alcohol consumption. A 15-year-old boy was healthy until 3 months ago. He has been having episodes of crampy abdominal pain and explosive liquid stools 2-3 times a day. He has had intermittent fevers and has noted blood in the stool. He has a documented 15-lb weight loss. Question What is the best way to start treatment? Answer Choices 1 Diphenoxylate hydrochloride with atropine sulfate 2 Eliminate lactose-containing foods 3 Loperamide HCL 4 Eliminate all high fiber foods 5 Sulfasalazine - ✔✔Sulfasalazine This patient has ulcerative colitis. Therefore, it would be most appropriate to start treatment with sulfasalazine at 50-75 mg/kg/day in 2-4 divided doses. Sulfasalazine is a sulfa drug that has important anti-inflammatory properties because it inhibits the synthesis of mediators of the inflammatory response. In some patients, sulfasalazine is poorly tolerated, in which case mesalamine (50-100 mg/kg/day) and balsalazide (110-175 mg/kg/day) are preferable treatments [Show More]
Last updated: 2 years ago
Preview 1 out of 348 pages
Buy this document to get the full access instantly
Instant Download Access after purchase
Buy NowInstant download
We Accept:
Comprises of exam Questions and answers, latest revisions for exams
By bundleHub Solution guider 2 years ago
$38
11
Can't find what you want? Try our AI powered Search
Connected school, study & course
About the document
Uploaded On
Aug 22, 2022
Number of pages
348
Written in
This document has been written for:
Uploaded
Aug 22, 2022
Downloads
0
Views
232
In Scholarfriends, a student can earn by offering help to other student. Students can help other students with materials by upploading their notes and earn money.
We're available through e-mail, Twitter, Facebook, and live chat.
FAQ
Questions? Leave a message!
Copyright © Scholarfriends · High quality services·